Ortaokul - Lise - ALTIN NOKTA · Endoplazmik Retikulumda sentezlenir C) Granüllü Endoplazmik...

34
Çözüm: Aliye ALTUĞ İZMİR - 2016 ALTIN NOKTA YAYINEVİ Bu kitaptaki soruların Yayın Hakkı TÜBİTAK (BAYG)’a ait olup, kurumun 22.11.2004 Tarihli B.02.1.TBT.0.06.03.00/1390-4015 sayılı yazılı izni ile yayımlanmıştır. Ulusal Biyoloji Olimpiyatı Soru ve Çözümleri Ortaokul - Lise

Transcript of Ortaokul - Lise - ALTIN NOKTA · Endoplazmik Retikulumda sentezlenir C) Granüllü Endoplazmik...

Page 1: Ortaokul - Lise - ALTIN NOKTA · Endoplazmik Retikulumda sentezlenir C) Granüllü Endoplazmik Retikulum yapısında yer alan proteinlerin sentezi GER’da gerçekleşir D) Sitoplazmada

Çözüm: Aliye ALTUĞ

İZMİR - 2016ALTIN NOKTA YAYINEVİ

Bu kitaptaki soruların Yayın Hakkı TÜBİTAK (BAYG)’a ait olup,

kurumun 22.11.2004 Tarihli B.02.1.TBT.0.06.03.00/1390-4015

sayılı yazılı izni ile yayımlanmıştır.

Ulusal Biyoloji OlimpiyatıSoru ve Çözümleri

Ortaokul - Lise

Page 2: Ortaokul - Lise - ALTIN NOKTA · Endoplazmik Retikulumda sentezlenir C) Granüllü Endoplazmik Retikulum yapısında yer alan proteinlerin sentezi GER’da gerçekleşir D) Sitoplazmada

Copyright©Altın Nokta Basım Yayın Dağıtım BilişimISBN 978-605-5255-03-9

Tübitak Soruları SerisiTübitak Ulusal Biyoloji Olimpiyatı Soru ve Çözümleri

Aliye ALTUĞ[email protected]

Bu kitabın her hakkı saklıdır. Tüm hakları Altın Nokta Basım Yayın Dağıtım'a aittir. Kısmen deolsa alıntı yapılamaz. Metin ve sorular, kitabı yayımlayan kurumun önceden izni olmaksızın elek-tronik, mekanik, fotokopi ya da herhangi bir kayıt sistemiyle çoğaltılamaz, yayımlanamaz.

Bu kitaptaki soruların Yayın Hakkı TÜBİTAK (BAYG)’a ait olup,kurumun 22.11.2004 Tarihli B.02.1.TBT.0.06.03.00/1390-4015

sayılı yazılı izni ile yayımlanmıştır.

Genel Yayın YönetmeniHalil İ. AKÇETİN

Yayın EditörüLeyla Gündoğdu

Kapak-DizgiAltın Nokta Dizgi-Grafik

MatbaaKanyılmaz Matbaa

Bornova / İzmir Tel: 0 (232) 449 14 43

Yayın - DağıtımAltın Nokta Basım Yayın Dağıtım859 Sk. No:1/Z-4 Konak / İZMİR

Tel- Fax : 0 (232) 441 25 95

www.nokta2000.comwww.altinnokta.com.tr

www.kitapana.com

[email protected]@altinnokta.com.tr

[email protected]

Basım Tarihi: Kasım - 20155. Basım

Page 3: Ortaokul - Lise - ALTIN NOKTA · Endoplazmik Retikulumda sentezlenir C) Granüllü Endoplazmik Retikulum yapısında yer alan proteinlerin sentezi GER’da gerçekleşir D) Sitoplazmada

Bu kitap neden yazıldı30 yıldır üniversite sınavlarına öğrenci hazırlayan ve sınav konularında yoğunla-şan, üniversitede edindiğim bilgilerin çoğunu kullanmadan ve yeni nesillere akta-ramadan meslek hayatımın sona ereceğini düşünerek üzülürken; “TÜBİTAKBiyoloji Olimpiyatları Sorularını çözermisiniz?” diye bir teklif aldığımda yıllardıristediğim bir işi yapma fırsatı bulduğumu düşündüm.

2000 yılından başlayan ve 2012 yılını da kapsayan soruları okuduğumda soru-ların yüzde ellisini bile araştırma yapmadan çözemeyeceğimi gördüm. Yerli veyabancı birçok kaynaktan araştırma yaparken, aradığım konuyu bulamadığımhalde başka konulara dalmak, bilgilerimi tazelemek ve yeni bilgiler öğrenmektüm yorgunluğumu unutturdu.

Elinizdeki kitap bu ürünleri daha çok insanın yararlanacağı bir kaynak halinegetirmeyi hedeflemektedir. Kitapta önce soru sonra yanıt gelecek şekilde birdüzenleme yapılmıştır. Sorudaki doğru yanıtın açıklanmasının yanı sıra kısacadiğer seçenekler hakkında da bilgi verilmiştir.

Bu kitabın hazırlanmasında kıymetli fikirlerinden, eleştirilerinden ve emeklerin-den yararlandığım Sevgili arkadaşlarım, Ayşe Avcı, Mülkiyet Şimşek, BehiceVarlı, Rıza Öztürk, Seher Arda, Çiğdem Yılmazbaş ve Feride Sezgin'e, kitabınbasımını üstlenen ALTIN NOKTA YAYINEVİ ekibine, bu kitabı onlardan çaldığımzamanlarda yazdığım sevgili aileme teşekkür etmeyi borç bilirim.

Gösterilen tüm özene karşın kitapta bazı hataların önüne geçilememiş olabilir.Bu hataların hoşgörüyle karşılanmasını ve eleştirilerin tarafıma bildirilmesinibekler, meslektaşlarıma ve öğrencilere yararlı olmasını dilerim.

Aliye ALTUĞBiyoloji Öğretmeni

[email protected]

Page 4: Ortaokul - Lise - ALTIN NOKTA · Endoplazmik Retikulumda sentezlenir C) Granüllü Endoplazmik Retikulum yapısında yer alan proteinlerin sentezi GER’da gerçekleşir D) Sitoplazmada

ULUSAL VE ULUSLARARASI OLİMPİYATLARAmacı, geleceğin bilim insanlarını yetiştirmek ve genç yeteneklerimizi teşvik etmek olarak tanımlanabilecek olan Olimpiyatetkinlikleri, TÜBİTAK Bilim İnsanı Destekleme Daire Başkanlığı (BİDEB) yürütülmektedir. Bu çalışmalar hem ulusal düzey-de hem de uluslararası düzeyde yapılmaktadır. Ulusal düzeyde gerçekleştirilen İlköğretim Matematik Olimpiyatı ile BilimOlimpiyatları sonuçlarına göre Ülkemizi Uluslararası yarışmalarda temsil edecek takımlar oluşturulmaktadır. Anılan yarış-malarla ilgili açıklayıcı bilgi Ulusal ve Uluslararası Olimpiyatlar başlıkları altında aşağıda verilmektedir.1- Ulusal Olimpiyatlar

a) Ulusal Ortaokul Matematik Olimpiyatıb) Ulusal Bilim Olimpiyatları

2- Uluslararası Olimpiyatlar:

1- Ulusal Olİmpİyatlar:a) Ulusal İlköğretim Matematik Olimpiyatıİlköğretim okullarının üst sınıflarına devam etmekte olan öğrencileri matematik alanında çalışmalar yapmak üzere yönlen-dirmek, ilgi ve yetenekleri doğrultusunda erken yaşlardan itibaren özel eğitim olanakları sağlayarak gelişmelerine katkıdabulunmak amacıyla her yıl ülke genelinde Ulusal İlköğretim Matematik Olimpiyatı düzenlenmektedir. 2011 yılında 28 il mer-keziyle KKTC’de gerçekleştirilen sınavlara 80 ilden 970 ilköğretim okulu başvurmuş ve 6456 öğrenci katılmıştır. İki aşama-lı olarak yapılan sınavlarda 34 öğrenci başarılı bulunarak altın, gümüş ve bronz madalyayla ödüllendirilmiştir. Bölgeseldüzeyde (7 bölge+KKTC) yapılan değerlendirme sonucuna göre başarılı bulunan öğrencilereyse bu başarılarını belgeleyensertifikalar verilmektedir. Bu sınavda öngörülen başarı düzeyini sağlayan öğrenciler Uluslararası Olimpiyatlara hazırlıkamaçlı kamplara çağrılarak bilgi düzeylerinin yükselmesine olanak sağlanmaktadır. Ulusal İlköğretim MatematikOlimpiyatı’nın bu yıl 17.’si yapılacaktır.

b) Ulusal Bilim OlimpiyatlarıOrtaöğretim kurumlarıyla (lise) ilköğretim kurumlarının 8. sınıflarına devam etmekte olan öğrencileri Temel Bilimlerde çalış-malar yapmaya özendirmek, çalışmalarını yönlendirmek ve bu alanlarda özel eğitim olanakları sağlamak yoluyla gelişme-lerine katkıda bulunmak amacıyla her yıl ülke genelinde MATEMATİK, FİZİK, KİMYA, BİYOLOJİ ve BİLGİSAYAR dalların-da Ulusal Bilim Olimpiyatları düzenlenmektedir. İlköğretim 8. sınıflar bu sınava FİZİK, KİMYA, BİYOLOJİ, BİLGİSAYAR dal-larında katılabilmektedir. Matematik dalında sınava katılmak isteyen ilköğretim öğrencileri için ayrıca Ulusal İlköğretimMatematik Olimpiyatı düzenlenmektedir. Mayıs ve Aralık aylarında, iki aşamalı olarak gerçekleştirilen bu yarışmaların 2011yılında 28 il merkezimiz ve KKTC’de yapılan ilk aşama sınavlarına 77 il ve 883 ortaöğretim okulundan toplam 10885 öğren-cimiz katılmıştır. Birinci Aşama Sınavlarında üstün başarı gösteren öğrencilerimiz yaz hazırlık kurslarında eğitilmekte, ayrı-ca her daldan yaklaşık 40-50 kişilik başarılı bir grup Aralık ayında yapılacak olan İkinci Aşama Sınavına davet edilmekte-dir. İkinci Aşama Sınavlarında dereceye girenlere madalya ve para ödülü verilmektedir. Bu öğrenciler, Uluslararası BilimOlimpiyatları için eğitilmek üzere kış hazırlık kurslarına çağrılmaktadır. Ayrıca, Türkiye genelinde dereceye giren öğrenciler,katıldıkları ilk Üniversite Giriş Sınavı’nda bir kereye özgü olmak üzere, aldıkları derece oranında ek katsayı uygulamasın-dan yararlanmaktadır. Bu sınavlarda derece alan ve lise öğrenimini bitirdiği yılın LYS sonucunda üniversitelerimizin temelbilim bölümlerinden birini kazanarak kayıt yaptıran öğrenciler, Bilim İnsanı Destekleme Daire Başkanlığı tarafından 2205-Yurt içi Lisans Burs Programı çerçevesinde desteklenmektedir. Ulusal Bilim Olimpiyatlarının bu yıl 20.’si yapılacaktır.

2- Uluslararası Olimpiyatlar:Uluslararası organizasyonlarca her yıl çeşitli dallarda bilimsel nitelikli Olimpiyatlar düzenlenmektedir. Ülkemizin deTÜBİTAK’ın organizasyonuyla katıldığı yarışmalar şunlardır: Uluslararası Bilgisayar (IOI), Biyoloji (IBO), Fizik (IPhO),Kimya (IChO), Matematik (IMO) Olimpiyatları ile Genç Balkan Matematik Olimpiyatı, Balkan Matematik Olimpiyatı veBalkan Bilgisayar Olimpiyatıdır. 1994 yılından beri katıldığımız ve bu yıl 23.’sü Tayland’da yapılacak olan IOI’ya 1999 yılın-da; 1993 yılından beri katıldığımız ve bu yıl 22.’si Çin’de yapılacak olan IBO’ya 2000 yılında; 1985 yılından beri katıldığı-mız ve bu yıl 42.’si Tayland’da yapılacak olan IphO’ya 2001 yılında; 1985 yılından beri katıldığımız ve bu yıl 52.’siHollanda’da yapılacak olan IMO’ya 1993 yılında; Ülkemiz ev sahipliği yapmıştır. 1994 yılından itibaren katıldığımız ve buyıl 43.’sü yapılacak olan IchO’ya Ülkemiz ev sahipliği yapacaktır. 1998 yılında 6. Balkan Bilgisayar Olimpiyatı, 2002 yılın-da 10. Balkan Matematik Olimpiyatı, 2003 yılında 7. Genç Balkan Matematik Olimpiyatı da yine ülkemizde düzenlenmiştir.Uluslararası Bilim Olimpiyatlarında Ülkemizi temsil edecek takımlar Ulusal Bilim Olimpiyatlarına katılmış ve kamplardabaşarılı olmuş öğrencilerin, çeşitli sınavlar sonucunda seçilmeleriyle oluşmaktadır. Şu ana kadar katıldığımız UluslararasıBilim Olimpiyatlarında Ülkemizi temsil eden öğrencilerimiz toplam 33 altın, 117 gümüş, 190 bronz ve 42 mansiyonkazanarak başarılarını kanıtlamışlar ve gurur kaynağımız olmuşlardır.Uluslararası Olimpiyatlarda derece alan öğrencilere plaket, para ödülü verilmekte, ayrıca olimpiyat takımlarında yer alarak üni-versitelerimizin temel fen, uygulamalı fen ve sağlık bilimlerinden birine kayıt olan öğrenciler, TÜBİTAK’ın üniversite lisans bur-siyeri olmaya hak kazanmaktadırlar. Ayrıca olimpiyat takımlarına seçilen öğrenciler LYS’ye girdikleri yıl, bir defaya mahsusolmak üzere uluslararası yarışmada aldıkları derecelerle orantılı ek katsayı uygulamasından, bu olimpiyatlarda altın, gümüşve bronz madalya kazananlarsa üniversitelere sınavsız yerleştirme olanaklarından yararlanabileceklerdir. (Bkz. ÖSYS kılavu-zu) Uluslararası Olimpiyatlarla ilgili her türlü kaynağa http://olympiads.win.tue.nl adresinden ulaşabilirsiniz.

Ulusal ve Uluslararası Olimpiyatlar

Page 5: Ortaokul - Lise - ALTIN NOKTA · Endoplazmik Retikulumda sentezlenir C) Granüllü Endoplazmik Retikulum yapısında yer alan proteinlerin sentezi GER’da gerçekleşir D) Sitoplazmada

a) Ulusal İlköğretim Matematik OlimpiyatıBAŞVURU KOŞULLARI (*)

• Matematik dalında yapılacak sınavlara, ülkemizdeki ve konuk statüsünde K.K.T.C.’deki ilköğretim okullarının 6, 7, 8. sınıflarındaokuyan başarılı öğrenciler arasından okul yönetimlerince seçilecek en çok 7 öğrenci katılabilecektir. Öğrencilerin T.C. vatandaşıolmaları gereklidir.

• Ulusal İlköğretim Matematik Olimpiyatına, başvurular Okul Müdürlükleri tarafından yapılacaktır. Bireysel başvurular kabul edil-memektedir.

BAŞVURU YÖNTEMİ• Ulusal İlköğretim Matematik Olimpiyatı iki aşamalı olarak yapılacaktır TÜBİTAK-BİDEB adına Birinci Aşama Sınavları MEB Yenilik

ve Eğitim Teknolojileri Genel Müdürlüğünce yürütülecek ve başvurular http://www.meb.gov.tr adresinden öğrencinin halen öğre-nim gördüğü kurum müdürlüğünce yapılacaktır.

• Okul Müdürlüklerinin sınava katılacak öğrencilerle ilgili istenilen bilgileri aşağıda verilen başvuru tarihleri arasındahttp://www.meb.gov.tr adresinde yer alan formları doldurarak girmeleri gerekmektedir.

• Programla ilgili ilan ve açıklamalar TÜBİTAK – BİDEB adına MEB Yenilik ve Eğitim Teknolojileri Genel Müdürlüğü tarafından tümilköğretim okullarına duyurulacak ve http://www.meb.gov.tr, http://www.tubitak.gov.tr web adreslerinde ilan edilecektir.

BAŞVURU ADRESİhttp://www.meb.gov.tr

SINAV TARİHİ VE YERİhttp://www.meb.gov.tr .

SINAV YÖNTEMİA) Birinci Aşama SınavıUlusal Ortaokul Matematik Olimpiyatı Birinci Aşama sınavı, çoktan seçmeli 32 sorudan oluşan test şeklindedir.Tüm öğren-ciler aynı soruları cevaplayacaklardır. (Sınıflara göre ayrı soru grubu yoktur.)

Değerlendirme: Değerlendirme bölgesel ve ülke genelinde olmak üzere iki kategoride yapılacaktır.a) Bölgesel Değerlendirme: Birinci Aşama Sınavı sonucuna göre, belli bir taban puanın üzerinde puan alan öğrenciler arasından,

bulundukları coğrafi bölge itibariyle (7 coğrafi bölge ve K.K.T.C.) derece kazananlara başarı belgesi verilir.b) Ülke Genelinde Değerlendirme: Türkiye genelinde kişisel başarı sıralaması dikkate alınarak, ilgili Komitece İkinci Aşama Sınavı’na

girmeye hak kazanan öğrenciler belirlenir.

İkinci Aşama Sınavı’na girmeye hak kazanan öğrenciler, 2012 yazında TÜBİTAK tarafından düzenlenecek olan YazOkuluna davet edilir. Birinci Aşama Sınavı sonunda, sınav sonuçları ve katılan tüm öğrencilerin puanları sınav tarihindensonra 1 ay içinde TÜBİTAK web sayfasında ilan edilir.

B) İkinci Aşama SınavıBirinci Aşama Sınavı sonucuna göre, İKİNCİ AŞAMA SINAVI’na katılmaya hak kazanan öğrencilerin sınavı Aralık 2012 için-de Ankara’da yapılacaktır. Sınav tek oturumda yazılı olarak gerçekleşecektir.

ÖDÜLLERİkinci Aşama Sınavları sonucunda dereceye giren 30 öğrenciye madalya ve başarı belgesi, madalya alan öğrencilerinöğretmenlerine ise takdir belgesi verilir.İlgili Komitece öngörülen düzeyde başarı gösterdiği belirlenen öğrenciler, 2012 - 2013 öğretim yılı yarıyıl tatilindeTÜBİTAK tarafından düzenlenecek Uluslararası Bilim Olimpiyatları Kış Okuluna davet edilir.

Madalya alan öğrencilerden Genç Balkan Matematik Olimpiyatı’na katılmak için yaşı uygun olanlar, 2013 ilkbaharın-da yapılacak olan Genç Balkan Matematik Olimpiyatı Takım Seçme Sınavı’na çağrılacaklardır. (Genç Balkan MatematikOlimpiyatı’na katılacak öğrencilerin, Olimpiyat sınavının yapılacağı gün 15,5 yaşını aşmamış olmaları gerekmektedir.)

Ulusal İlköğretim Matematik Olimpiyatında, bilgi içeriği açısından ilköğretim 8. sınıfları esas alınmakta olup, zorluk düzeyi,matematikte özel yetenekleri keşfetme amacı doğrultusunda belirlenmektedir. Bu nedenle, olimpiyat sınavlarının, gerek içerik,gerekse düzey bakımından, ilköğretim öğrencilerinin girdiği diğer seçme ve giriş sınavlarından farklı olduğu bilinmelidir.Ulusal İlköğretim Matematik Olimpiyatı, ilköğretim okullarının 6., 7. ve 8. sınıflarına devam etmekte olan tüm öğrencilereaçık olmakla birlikte, öğrencilerimizi istenmeyen türden bir basarisizlik duygusuyla karsı karsıya bırakmamak için, başvuru-larda bu içerik ve düzey farklılığının göz önünde bulundurulmasında yarar görülmektedir. TÜBİTAK – Bilim İnsanıDestekleme Daire Başkanlığı’nın sitesinde bulunan geçmiş sınav örneklerinin öğrenciler tarafından incelenmesinin sağlan-ması, bu açıdan yol gösterici olacaktır.

Page 6: Ortaokul - Lise - ALTIN NOKTA · Endoplazmik Retikulumda sentezlenir C) Granüllü Endoplazmik Retikulum yapısında yer alan proteinlerin sentezi GER’da gerçekleşir D) Sitoplazmada

b) Ulusal Bilim OlimpiyatlarıBAŞVURU KOŞULLARI(*)

• Matematik, Fizik, Kimya, Biyoloji ve Bilgisayar dallarında yapılacak sınavlara, ülkemizdeki ve konuk statüsünde olmak üzereK.K.T.C.’deki her ortaöğretim kurumu, başarılı öğrencileri arasından okul yönetimince seçilecek her dalda en çok 8’er öğrenci ile 2011yılında ikinci aşama sınavlarına katılmış olan öğrencileri için başvuru yapabilir. İlköğretim Kurumları da bu sınavlara, Matematik dışın-da kalan dallarda (Fizik, Kimya, Biyoloji ve Bilgisayar) 8. sınıfa devam etmekte olan başarılı öğrencileri arasından okul yönetiminceseçilecek en çok 2 öğrenci ile katılabilirler. Öğrencilerin T.C. vatandaşı olmaları gereklidir.

• Bu sınavlara katılmak üzere Matematik, Fizik, Kimya, Biyoloji ve Bilgisayar dallarından önerilecek ortaöğretim öğrencilerinin, 2011 –2012 öğretim yılında (2012’de) ortaöğretiminin son yılında olmamaları gerekmektedir. Matematik dalında önerilecek öğrencilerin enaz 3’ü ortaöğretim eğitiminin 1. veya 2. yılında olmalıdır.

BAŞVURU YÖNTEMİ• TÜBİTAK – BİDEB adına 20. Ulusal Bilim Olimpiyatları Birinci Aşama Sınavları 2012 yılında MEB Yenilik ve Eğitim Teknolojileri Genel

Müdürlüğünce yürütülecek ve başvurular http://www.meb.gov.tr adresinden öğrencinin halen öğrenim gördüğü kurum müdürlüğünceyapılacaktır. Bireysel başvurular kabul edilmeyecektir.

• Okul müdürlüklerinin sınava katılacak öğrencilerle ilgili istenilen bilgileri aşağıda verilen başvuru tarihleri arasındahttp://www.meb.gov.tr adresinden gerekli formları doldurarak girmeleri gerekmektedir.

• Programla ilgili ilan ve açıklamalar TÜBİTAK - BİDEB adına MEB Yenilik ve Eğitim Teknolojileri Genel Müdürlüğü tarafından tüm okul-lara duyurulacak ve http://www.meb.gov.tr http://www.tubitak.gov.tr/bideb web adreslerinde ilan edilecektir.

• Bir öğrenci en çok iki dalda önerilebilir. Başvuru girişi yapıldıktan sonra bir öğrencinin yerine başka birinin önerilmesi ya da daha önce8’den az öğrenci önerilmiş olan dallarda yeni öğrenciler önerilerek sayının tamamlanması gibi değişiklik istemleri kabul edilmez.

BAŞVURU ADRESİhttp://www.meb.gov.trSINAV TARİHİ VE YERİhttp://www.meb.gov.tr

SINAV YÖNTEMİA) Birinci Aşama SınavlarıSınavlarda;

a) Çoktan seçmeli sorulardan oluşan testler uygulanır.b) Tüm öğrenciler aynı soru grubunu yanıtlar.(Sınıflara göre ayrı soru grubu yoktur)c) Fizik dalında, problemlerin çözümleri de dikkate alınarak değerlendirme yapılır.

d) Bilgisayar dalındaki sınavda temel bilgisayar, temel C programlama dili ve genel yetenek soruları yer alır.

Değerlendirme: Bütün dallarda değerlendirmeler bölgesel ve ülke genelinde olmak üzere iki kategoride yapılır.a) Bölgesel Değerlendirme• Sınav sonucuna göre bütün dallarda, belli bir taban puanın üzerinde puan alan öğrenciler arasından, bulundukları coğrafi bölge itibariy-le (7 coğrafi bölge ve K.K.T.C.) derece kazananlara başarı belgesi verilir.

b) Ülke Genelinde Değerlendirme• Bütün dallarda, Türkiye genelinde kişisel başarı sıralaması dikkate alınarak, ilgili Komitelerce İkinci Aşama Sınavları’na girmeye hakkazanan öğrenciler belirlenir. Bu öğrencilerin öğretmenlerine takdir belgesi verilir.• İkinci Aşama Sınavları’na girmeye hak kazanan öğrenciler aynı zamanda 2012 yazında TÜBİTAK tarafından düzenlenecek olan YazHazırlık Okuluna davet edilir. Birinci Aşama Sınavları sonunda, sınav sonuçları ve katılan tüm öğrencilerin puanları sınav tarihinden sonra 1ay içinde TÜBİTAK web sayfasında ilan edilir.

B) İkinci Aşama Sınavları• Birinci Aşama Sınavları sonucuna göre, Türkiye geneli dikkate alınarak yapılan kişisel değerlendirmeler sonunda İKİNCİ AŞAMASINAVLARI’na katılmaya hak kazanan öğrencilerin sınavı Aralık 2012 içinde Ankara’da yapılacaktır.• İkinci Aşama Sınavları Matematik ve Biyoloji dallarında yazılı; Fizik ve Kimya dallarında yazılı ve uygulamalı; Bilgisayar dalında ise uygu-lamalı yapılır.• İlgili Komitelerce öngörülen düzeyde başarı gösterdiği belirlenen öğrenciler, 2012 - 2013 öğretim yılı yarıyıl tatilinde TÜBİTAK tarafın-dan düzenlenecek Uluslararası Bilim Olimpiyatları Kış Hazırlık Okuluna davet edilir. Kış Hazırlık Okulu sonunda başarı gösteren öğrencilerUluslararası Bilim Olimpiyatları ekiplerine seçilir. (Bu süreç, ekipte yer alan öğrencilerin özel yetiştirme kurslarında eğitilmeleri, kesin ekiple-rin Nisan, Mayıs ve Haziran aylarında üç dönem kursa alınmaları ve Temmuz ayından itibaren Olimpiyatların yapılacağı ülkelere gönderil-meleri şeklinde yaklaşık 6 ay süren etkinlikleri kapsamaktadır.)

ÖDÜL VE KAZANIMLAR• İkinci Aşama Sınavları sonucunda dereceye giren öğrencilere TÜBİTAK tarafından para ödülü, başarı belgesi ve madalya verilir. Türkiyegenelinde dereceye giren öğrenciler, katıldıkları ilk Üniversite Giriş Sınavı’nda bir defaya özgü olmak üzere, aldıkları derece oranında ek kat-sayı uygulamasından yararlanırlar (Bkz. ÖSYS Kılavuzu). Ortaöğretim öğrenimini bitirdikten sonra gireceği ilk lisans yerleştirme sınavı (LYS)sonucunda üniversitelerimizin bu yarışma alanlarıyla ilgili bölümlerinden/programlarından birini kazanarak kayıt yaptıran (bu alanlarla ilgilibölümler/programlar her yıl ÖSYS Kılavuzu’nda yer almaktadır ) öğrenciler 2205 – Yurt İçi Lisans Burs Programı kapsamında desteklenirler.• Uluslararası Olimpiyatlarda madalya alan öğrencilere TÜBİTAK tarafından ödüller verilmekte ve ekipte yer alan tüm öğrencilerTÜBİTAK’ın üniversite lisans bursiyeri olmaya hak kazanmaktadır. Bu öğrencilerin üniversiteye girdikleri yıl burs için TÜBİTAK’a başvurma-ları gerekmektedir. Ayrıca 2001 yılından itibaren yürürlüğe konan bir kanun ile bu öğrencilerden Uluslararası yarışmada madalya kazanan-lara Yükseköğretim Giriş Sınavı’na girmeden alanlarındaki yükseköğretim programlarından istediklerine (yükseköğretim kurumlarından burs-lu olanlar hariç) kayıt yaptırabilme hakkı verilmektedir. Sınava katılacak öğrencilerin sınav içeriği ile ilgili bilgi edinebilmeleri için TÜBİTAKweb sitesindeki soruları incelemeleri önerilmektedir.

Page 7: Ortaokul - Lise - ALTIN NOKTA · Endoplazmik Retikulumda sentezlenir C) Granüllü Endoplazmik Retikulum yapısında yer alan proteinlerin sentezi GER’da gerçekleşir D) Sitoplazmada

İÇİN

DEK

İLER

TÜBİTAK ULUSAL BİYOLOJİ OLİMPİYATI1. AŞAMA SORU ve ÇÖZÜMLERİ

İlköğretim - Ortaöğretim

8. Ulusal Biyoloji Olimpiyatı Soru ve Çözümleri İlköğretim - Ortaöğretim 2000 ........... 9

9. Ulusal Biyoloji Olimpiyatı Soru ve Çözümleri İlköğretim - Ortaöğretim 2001 ........... 37

10. Ulusal Biyoloji Olimpiyatı Soru ve Çözümleri İlköğretim - Ortaöğretim 2002 ........... 63

11. Ulusal Biyoloji Olimpiyatı Soru ve Çözümleri İlköğretim - Ortaöğretim 2003 ........... 91

12. Ulusal Biyoloji Olimpiyatı Soru ve Çözümleri İlköğretim - Ortaöğretim 2004 ........... 119

13. Ulusal Biyoloji Olimpiyatı Soru ve Çözümleri İlköğretim - Ortaöğretim 2005 ........... 145

14. Ulusal Biyoloji Olimpiyatı Soru ve Çözümleri İlköğretim - Ortaöğretim 2006 ........... 173

15. Ulusal Biyoloji Olimpiyatı Soru ve Çözümleri İlköğretim - Ortaöğretim 2007 ........... 201

16. Ulusal Biyoloji Olimpiyatı Soru ve Çözümleri İlköğretim - Ortaöğretim 2008 ........... 227

17. Ulusal Biyoloji Olimpiyatı Soru ve Çözümleri İlköğretim - Ortaöğretim 2009 ........... 253

18. Ulusal Biyoloji Olimpiyatı Soru ve Çözümleri İlköğretim - Ortaöğretim 2010 ........... 279

19. Ulusal Biyoloji Olimpiyatı Soru ve Çözümleri İlköğretim - Ortaöğretim 2011 ........... 307

20.Ulusal Biyoloji Olimpiyatı Soru ve Çözümleri İlköğretim - Ortaöğretim 2012 ........... 335

Page 8: Ortaokul - Lise - ALTIN NOKTA · Endoplazmik Retikulumda sentezlenir C) Granüllü Endoplazmik Retikulum yapısında yer alan proteinlerin sentezi GER’da gerçekleşir D) Sitoplazmada

ALTIN NOKTA YAYINLARI:

Kalite ve içeriği, ticaretin önüne koyarak etkili ve ilkeli yayınlar çıkarmayı amaç

edinmiştir. Yayınevimiz yalın dili ile anlaşılır, deneyimli öğretmenleri ile güvenilir yayınlar sunmaktadır.

Yayınlarımızda, konu kısıtlamasına gitmeden, tüm konular işlenmeyeçalışılmıştır. Ezberci öğretim yerine yaratıcılık ön plana çıkarılmıştır. Yayınlarımız araştırma yapan eğitimci-lerimiz için dolu dolu bir başvuru kaynağı olacak biçimde hazırlanmıştır.

Ülkemizin en büyük sorunlarından birisi de bilim adamı yetiştirme konusundayetersiz kalmamız, yetiştirdiğimiz bilim adamlarını hak ettiği konumlara getiremeyişimizdir. Yayınevi olarakamacımız bu konuda öğretmen ve öğrencilerimize etkili yayınlar hazırlamak. Edindiğimiz bilgiyi paylaşmak,özellikle TÜBİTAK olimpiyat sınavlarına hazırlanan öğrencilerimize kaynaklar sunmaktır. Kısıtlı bütçemizle,gönüllü öğretmenlerimizle bilimsel yayınlar çıkarmayı amaç edindik.

Güzel ülkemizin her yerinden gönülleriyle destek veren öğretmenlerimize,yayınlarımızın hazırlanışında emeğini hiçbir zaman esirgemeyen eşim Leyla AKÇETİN’e, yazarlarımız AliyeAltuğ’a, Ramazan Karakale’ye ve Yüksel Öztaş’a, sonsuz teşekürlerimi sunarım.

Halil İ. AKÇETİNGenel Yayın Yönetmeni

ULUSLARARASI BİLİM OLİMPİYATLARI

Bilgisayar, biyoloji, matematik, fizik ve kimya dallarında Uluslararası Bilim Olimpiyatları düzenlenmektedir. 2012 yılında,Bilgisayar dalında İtalya’da, Biyoloji dalında Singapur’da, Fizik dalında Estonya’da, Kimya dalında A.B.D.’de ve Matematikdalında Arjantin’de yapılacak olan Uluslararası Bilim Olimpiyatları'na katılacak Türk takımları 19. Ulusal BilimOlimpiyatlarına katılmış ve başarılı olmuş öğrenciler arasından seçilecektir. Uluslararası olimpiyatlarda derece alan öğren-cilere para ödülü verilmekte, olimpiyat takımlarında yer alarak üniversitelerimizin Doğa Bilimleri, Mühendislik ve Teknoloji,Tıbbi Bilimler ve Tarımsal Bilimlerden birine kayıt olan öğrenciler, TÜBİTAK'ın üniversite lisans bursiyeri olmaya hak kazan-maktadır. Ayrıca olimpiyat takımlarına seçilen öğrenciler YGS'ye girdikleri yıl, bir defaya mahsus olmak üzere uluslararasıyarışmada aldıkları derecelerle orantılı ek katsayı uygulamasından; bu olimpiyatlarda altın, gümüş veya bronz madalyakazananlar ise üniversitelere sınavsız yerleştirilme olanaklarından yararlanabilecektir.

Bu programlarda derece alan öğrenciler, ilgili koşulları sağladıkları takdirde, YGS'de ek puan alabileceklerdir.(Bkz: 2012 ÖSYS Öğrenci Seçme ve Yerleştirme Sistemi Kılavuzu)

TÜBİTAK-BİDEBTunus Caddesi No: 80, 06100 Kavaklıdere-ANKARA

Tel: 0-312-468 53 00/3811 - 3824 - 3819E-mail adresi: [email protected]

[email protected]@tubitak.gov.tr

Page 9: Ortaokul - Lise - ALTIN NOKTA · Endoplazmik Retikulumda sentezlenir C) Granüllü Endoplazmik Retikulum yapısında yer alan proteinlerin sentezi GER’da gerçekleşir D) Sitoplazmada

335

alt

ın n

ok

ta y

ay

ınla

rı ©

2012 - 20. Ulusal Biyoloji Olimpiyatı 1. Aşama Soru ve Çözümleri

1. Aşağıdakilerden hangisi Granüllü Endoplaz-mik Retikulumda (GER) sentezlenen bir pro-tein için doğru değildir?

A) Hücreden bir salgı ürünü olarak salgılanır

B) Hücre zarı yapısına katılacak proteinler GranüllüEndoplazmik Retikulumda sentezlenir

C) Granüllü Endoplazmik Retikulum yapısında yeralan proteinlerin sentezi GER’da gerçekleşir

D) Sitoplazmada işlev görecek enzim bu yolla karşı-lanır

E) Düz Endoplazmik Retikulum yapısında yer alanproteinler GER’da sentezlenir

2. Peroksizomlar, hayvan hücrelerinde bulunanlizozomlar gibi, enzimler içeren bir organeldir.Sahip olduğu enzimler sayesinde bazı reaksi-yonları gerçekleştirir. Bu bağlamda peroksi-zomda yer alan enzim işlevleri ve peroksi-zomla ilgili olarak aşağıdaki ifadelerden han-gisi yanlıştır?

A) Peroksizomlarda yan ürün olarak hidrojen peroksitoluşur.

B) Bazı peroksizomlarda büyük yağ asitlerinin oksi-jenle yıkımı gerçekleştirilir.

C) Karaciğer hücreleri gibi bazı hücrelerde detoksifi-kasyona katılırlar.

D) Hidrojen peroksiti suya dönüştüren enzimlerperoksizomlarda yer alır.

E) Yeni peroksizomlar golgiden tomurcuklanarak oluşur.

3. Çift tabakalı lipit zardan gradiyent farkına bağlıolarak polar bir molekül taşınımı kimyasal enerjikullanılmadan gerçekleştiriliyor.

Bu durumda molekülün taşınımında aşağıda-ki yollardan hangisi kullanılmıştır?

A) Basit difüzyon B) Simport C) Ozmoz

D) Kolaylaştırılmış difüzyon E) Uniport

ÇÖZÜM:

Simport sistem iki farklı molekülü aynı yönde hareketettiren sistemdir. Bakterilerdeki proton ve şeker trans-portörleri, Na+ ve amino asit transportörleri simporttransportörleridir. Kolaylaştırılmış difüzyonda memb-ranlardan geçiş başlıca üç şekilde gerçekleşir.

1. Membranı tümüyle kateden ve porinler diye adlan-dırılan kanat proteinleri vasıtasıyla;

2. Molekül ağırlığı ve tipine göre farklı olan taşıyıcıproteinler vasıtasıyla

3. Proteinlerin glip glop hareketi vasıtasıyla. Bu sıradaenerji harcanmaz. Olay gradient farkına bağlı olarakgerçekleşir.

Uniport geçiş: Zarda bulunan taşıyıcı moleküllerlekonsantrasyon farkına bağlı olarak birim zamanda yal-nızca bir molekül ya da iyonun sitozolden içeri ya dadışarı taşınmasıdır. Memeli hücrelerinde glikozuntaşınması bu şekilde gerçekleşir. Basit difüzyon çokyoğun ortamdan az yoğun ortama enerji harcanmadanmadde aktarılmasıdır. Suyun difüzyonu ise osmozdur.

ÇÖZÜM:

Granüllü endoplazmik retikulum üzerindeki ribozomlar-da hücrenin zar proteinleri ve harici proteinleri sentez-lenir. Granüllü ER protein ve fosfolipitler ekleyerekhücre zarının büyütür. Zar fosfolipitlerini ve salgı prote-inlerini de sentezler. Sitoplazmada işlev görecekenzimler ise sitoplazmadaki serbest ribozomlarda sen-tezlenir.

ÇÖZÜM:

Peroksizomlar çeşitli substratlardan oksijene hidrojenaktararak yan ürün olarak hidrojen peroksit oluşturanenzimler içerir. Bazı peroksizomlar yağ asitlerini dahaküçük moleküllere yıkmak için oksijen kullanır. Oluşanküçük moleküller mitokondrilere gönderilir.

Karaciğerdeki peroksizomlar zehirlerden oksijene hid-rojen aktarırlar. (detoksifikasyon) Ayrıca bu organelH2O2 yi suya dönüştüren enzimler içerir. Glioksizomadı verilen peroksizomlar bitki tohumlarının yağ depo-layan dokularında yer alır. Bu organel protein ve lipitle-rin sitozolde bir araya gelmesiyle oluşur. Belirli birbüyüklüğe ulaştığında ikiye bölünür ve sayıca artar.

4. Hücre yüzeyinde yer alan mikrovilluslarlailgili olarak aşağıdaki ifadelerden hangisiyanlıştır?

A) Geri emilimin gerçekleştirildiği hücrelerde bulunur

B) Hareketleri sayesinde bulunduğu hücre yüzeyin-den maksimum madde hareketi sağlanır

C) Yapısını aktin filamentler oluşturur

D) Hücre yüzeyinde birbirine paralel düzenli uzantı-lar halinde yer alır

E) Hücre apikalinin kasılmasıyla mikrovillusların ara-sına daha fazla içerik girmesi sağlanır

Page 10: Ortaokul - Lise - ALTIN NOKTA · Endoplazmik Retikulumda sentezlenir C) Granüllü Endoplazmik Retikulum yapısında yer alan proteinlerin sentezi GER’da gerçekleşir D) Sitoplazmada

336

2012 - 20. Ulusal Biyoloji Olimpiyatı 1. Aşama Soruları ve Çözümleri

alt

ın n

ok

ta y

ay

ınla

rı ©

5. Hücre iskeleti elemanlarıyla ilgili olarak aşa-ğıdaki ifadelerden hangisi veya hangileriyanlıştır?

I. Mikrofilament, mikrotübül ve ara filamentler hüc-renin ihtiyacına göre yıkılıp yeniden yapılabilirler

II. Mikrofilamentler endositoz, ekzositoz gibi hücrezarına bağlı işlevlerde rol oynarlar

III. Mikrotübüller hücre içi taşınımda işlev görürler

IV. Ara filamentler organellerin hücre içinde yerleri-nin sabitlenmesinde görev alırlar

A) Yalnız I B) I ve II C) II ve III

D) Yalnız IV E) I, II ve IV

ÇÖZÜM:

Taşıyıcı proteinler

Hücre zarında yerleşmiş olan bir kısım proteinler taşı-yıcı olarak davranırlar. Taşıyıcı proteinler hücre içinenelerin girip çıktığını düzenlemeye yardımcı olurlar. Buproteinlerin de iki özel kısmı bulunur: Hücre zarı mal-zemesine bağlanan yağ dostu kısım ve hücre zarındantaşınması gereken maddelere bağlanan diğer kısım.Taşıyıcı protein taşınacak maddeye bağlanarak, rota-sını değiştirir ve yükünü hücre zarı boyunca taşır.

Taşıyıcı proteinler belirli moleküllere yapışırlar vesadece bunları hücre içine getirirler. Bu görevleriniyerine getirirken şekillerinde bir değişiklik olur vebazen maddeleri hücre zarından geçirmek için enerji-ye ihtiyaç duyarlar. Hücre zarının kendisinde delikleryoktur. Bu yüzden lipit çift katlı tabakadan oluşanhücre zarından direkt olarak geçemeyen su, protein,nükleik asit ve bazı küçük moleküller, bu “taşıyıcı” pro-teinler yoluyla hücre içine girerler.

Taşıyıcı proteinlerin üç boyutlu amino asit dizilimlerinedeniyle, küçük bir dar geçit yapmaları kolay olur.Böylece bu boşluğa sığan belirli büyüklükteki madde-lerin bu kanaldan geçmesi mümkün olur. Ancak sade-ce büyüklük burada geçiş için yeterli değildir; hücrezarı sadece hücrenin ihtiyacı olan, içeri alınması gere-ken maddeleri alarak seçici-geçirgin bir özellik sergiler.

Taşıyıcı proteinler taşınacak moleküle özgüldürler.

6. Taşıyıcı proteinlerle ilgili olarak aşağıdaki ifa-delerden hangisi yanlıştır?

A) Bazı taşıyıcı proteinler belirli bir molekülün geçişi-ni sağlayan hidrofilik kanal işlevi görür

B) Bazı taşıyıcı proteinler iyon yada molekülleri fizik-sel olarak taşırlar

C) Polar moleküller taşıyıcı proteinler aracılığıylalipid zardan taşınır

D) Taşıyıcı proteinler integral proteinlerdendir

E) Taşıyıcı proteinler taşınacak moleküle özgül değil-dirler

7. Aşağıdaki seçeneklerin hangisinde “organel-işlev” eşleştirmesi yanlış olarak verilmiştir?

A) Çekirdek - DNA replikasyonu

B) Mitokondri - ATP eldesi

C) Lizozom - Bakterilerin inaktivasyonu

D) Düz endoplazmik retikulum - Polipeptit sentezi

E) Kloroplast - Işık enerjisinin şekerlerdeki kimyasalbağ enerjisine dönüştürülmesi

ÇÖZÜM:

Hücre iskeletini kuran üç temel lif tipinin en kalınımikrotübüllerdir. Aktin flamentleri denilen mikrofla-mentler ise en inceleridir. İntermediyer flamentler ortakalınlıktadır. Mikrotübüller hücreye biçim verip onudesteklerken motor moleküllerin hareket etmesini sağ-layan yollar olarak da görev yaparlar. Örneğin; salgıveziküllerini golgiden hücre zarına taşırlar.

Mikroflamentler hücre biçiminin korunmasında, kaskasılması, hücre hareketi, yalancı ayak oluşmasındada görev yaparlar. İntermediyer flamentler hücre biçi-minin korunmasında ve organellerin sabitlenmesindegörev yaparlar.

ÇÖZÜM:

Mikrovilluslar

Boyları yaklaşık, 0,6-0,8 mikron uzunluğunda, 0,08-0,1mikron kalınlığındadır. Özellikle bağırsak epitelindebulunan mikrovilluslar (çoğulu mikrovilli) yapılarında,makromolekülleri parçalayarak ve hücre içine taşıya-cak enzimleri bulundururlar. Silindirik ya da kübik epi-tel (örtü) hücrelerinin üst yüzeylerinde emme yüzeyinigenişletmek için hücrenin sitoplazmasından dışarıdoğru yaptığı uzantılardır. İçinde aktin filamentlerindenoluşmuş göbek proteinleri bulunur. Böcek Malpighitüpü hücreleri gibi bazı özel hücrelerde ise içinde mito-kondrilerin de bulunduğu parmak şeklindeki katlanma-lardır. Böbrek hücrelerinde fırça kenar yapısını, bağır-sak epitelinde çizgili kenar yapısını oluşturur.

Yapısı:

Genel olarak iki yapısal elemandan oluşur.

1. Aktin filamentleri

2. Bağlayıcı proteinler

Bağlayıcı proteinler de iki çeşittir;

1. Aktin filamentlerini birbirine bağlayan proteinler (fas-cin, fimbrin, spectrin)

2. Aktin filamentlerini hücre zarına bağlayan proteinler(miyozin 1)

Emilme için yüzey genişletirler fakat hareketi sağla-mazlar.

Yanıt: E

Page 11: Ortaokul - Lise - ALTIN NOKTA · Endoplazmik Retikulumda sentezlenir C) Granüllü Endoplazmik Retikulum yapısında yer alan proteinlerin sentezi GER’da gerçekleşir D) Sitoplazmada

337

2012 - 20. Ulusal Biyoloji Olimpiyatı 1. Aşama Soruları ve Çözümleri

alt

ın n

ok

ta y

ay

ınla

rı ©

ÇÖZÜM:

- Ökaryotik bir hücrede DNA nın eşlenmesi çekir-dekte gerçekleşir.

- Mitokondri hücrenin enerji santralidir.

- Kloroplast ışık enerjisini ATP ye, onu da kimyasalbağ enerjisine çevirir.

- Lizozom hücre içi sindirim enzimleri taşır.

- Düz endoplazmik retikulumda ribozom olmadığın-dan protein sentezi yapılmaz.

ÇÖZÜM:

Kirezin; hücre içine çekirdekten gelen kargoyu mikro-tübüller üzerine taşır.

Tübülin; hücre bölünmesi ve hücre hareketi gibi fonk-siyonlarda görev alan mikrotübüllerin önemli yapıtaşla-rından biridir. Tübülinlerin birbirine bağlanması ilehücre içinde uzun ipliksi yapılar oluşur. Hücre hareke-ti, bu uzun ipliksi yapılar sayesinde gerçekleşir.

Miyosin; kas kasılmasından sorumludur.

Aktin; kas kasılmasından sorumludur.

Dinein; mikrotübüller üzerinden kargonun çekirdeğinedoğru taşınmasını sağlar.

ÇÖZÜM:

Çekirdekçikte özel bir RNA tipi olan ribozomal RNAsentezlenir. Ribozomal RNA sitoplazmadan gelen pro-teinlerle bir araya getirilerek ribozomların temel birim-leri olan ribozom alt birimleri oluşturulur. Bu alt birimlerdaha sonra nüklear porlardan çıkarak sitoplazmayageçer ve ribozomları oluşturacak şekilde birleşirler.

8. Bir hücrede fazla miktarda Golgi Kompleksibirikiminin gözlenmesi aşağıdaki olaylardanesasen hangisi ile ilgili olabilir?

A) DNA replikasyon hızının artması

B) Protein sentez hızının artması

C) Fotosentez hızının artması

D) Lipit sentez hızının artması

E) Fazla miktarda madde salgılanması

9. Hücre dışında işlev görecek bir protein, hücreiçerisinde sentezlenerek hücre dışına salgılanır.Bu proteinin sentezinden itibaren salgılan-masına kadar olan süreçte iş gören molekül-ler aşağıdakilerden hangisidir?

A) Kinezin – Tübülin B) Dinein – Tübülin

C) Kinezin - Aktin D) Dinein – Aktin

E) Kinezin - Miyozin

10. Ökaryotik bir hücrede işlevsel bir ribozomunyapısı ve oluşumuyla ilgili olarak aşağıdaki-lerden hangisi yanlıştır?

A) Ribozom iki alt birimden oluşur

B) Alt birimlerin yapısında rRNA ve proteinler yer alır

C) İki alt birim çekirdekte birleşerek ribozom oluşur

D) Ribozom yapısına katılacak proteinler sitoplazma-dan temin edilir

E) Ribozom alt birimleri çekirdekçikte yapılır

ÇÖZÜM:

E.R dan ayrılan transport veziküllerinin çoğu golgiaygıtına gelir. Golgi bir üretim, depolama, ayırma vegönderme merkezidir. E.R. ürünleri burada değişikliğeuğratılır, depolanır ve daha sonra gidecekleri yerleregönderilir. Golgi aygıtı salgı için özelleşmiş hücrelerdebol miktarda bulunur. Yağ senteziyle de ilgilidir.

ÇÖZÜM:

Metabolizma sonucu oluşan azotlu bir artık olan amon-yak zehirlidir ve dışarı atılması gerekir. Bazı hayvanlarsu kaybını azaltmak amacıyla amonyağı üreye çevire-rek atarlar. Amonyak, karaciğerde ornitin devri denilenbir reaksiyonlar zinciriyle üreye çevrilir. Arjinin ornitindevrinde görev alan bir amino asit olduğundan diyet-ten tamamen çıkarıldığında vücutta aşırı miktardaamonyak artışı olur.

11. Aşağıdaki aminoasitlerden hangisi diyetten ta-mamen çıkarıldığında deney hayvanlarının ka-nında amonyağın aşırı artışına neden olabilir?

A) Lizin B) Histidin C) Arjinin

D) Asparajin E) Alanin

ÇÖZÜM:

Glikozda oluşan piruvat laktat dehidrogenaz enzimitarafından laktata çevrilir.

Piruvat+NADH Laktat + NADLaktatDehidrogenaz⎯ →⎯⎯⎯⎯⎯⎯⎯

12. Hızlı koşu sırasında kaslarda fazla miktardaLaktat oluşur ve ağrıya neden olur.Aşağıdakilerden hangisi Laktata dönüşümreaksiyonunu katalizleyen Laktat dehidroge-naz enziminin substratıdır?

A) Etanol B) Glukoz C) Piruvat

D) Fosfoenolpiruvat E) Laktik asit

Yanıt: E

Page 12: Ortaokul - Lise - ALTIN NOKTA · Endoplazmik Retikulumda sentezlenir C) Granüllü Endoplazmik Retikulum yapısında yer alan proteinlerin sentezi GER’da gerçekleşir D) Sitoplazmada

338

2012 - 20. Ulusal Biyoloji Olimpiyatı 1. Aşama Soruları ve Çözümleri

alt

ın n

ok

ta y

ay

ınla

rı ©

13. Aşağıdaki yapılardan hangisinde yağlardankarbohidrat sentezi gerçekleşir?

A) Lizozom B) Glioksizom

C) Endoplazmik retikulum D) Golgi

E) Hücre zarı

14. Histonlar, DNA yapısındaki fosfat gruplarıyla bağyapan proteinlerdir.

Aşağıdaki amino asitlerden hangisi histon-larda yüksek oranda bulunur?

A) Aspartik asit B) Alanin C) Glisin

D) Lizin E) Valin

ÇÖZÜM:

Bir molekülden karbondioksidin uzaklaştırılmasınadekarboksilasyon denir.

Krebs devrinde α ketoglutaratın süksinil CoA yadönüşmesi sırasında oksidatif dekarboksilasyon olur.

15. Aşağıdakilerden hangisi oksidatif dekarbok-silasyon reaksiyonuna örnektir?

A) Piruvatın Laktata dönüşümü

B) Piruvatın Oksalasetata dönüşümü

C) Süksinatın Fumarata dönüşümü

D) Malatın Oksalasetata dönüşümü

E) α-Ketoglutaratın Süksinil-CoA’ya dönüşümü

ÇÖZÜM:

Karbondioksit azalması, oksihemoglobin dissosiasyoneğrisini sola kaydırır. Bu durum;

→ Hemoglobinin oksijene olan ilgisinin artmasınaneden olur.

→ Dokulara oksijen bırakılması azalır.

→ Herhangi bir pO2’de hemoglobinin oksijen ile doy-gunluğu artar.

→ Oksihemoglobinin dissasiasyon eğrisinde solakayma, dokular tarafından alınan oksijen miktarınıazaltır.

Oksihemoglobinin dissasiasyon eğrisinde sola kaymaşu durumlarda gözlenir;

1) Karbondioksit kısmi basıncında azalma

2) H+ iyonları konsantrasyonunda azalma veya pH’daartma ⇒ pH 7,6

3) Isıda azalma

4) 2,3-Difosfogliserat düzeyinde düşme

5) HbF düzeyinde artma.

ÇÖZÜM:

Histon proteinleri ökaryotik kromatinlerde ilk düzeydeDNA paketlenmesinden sorumludur. Kromotindeki his-ton kütlesi DNA kütlesine eşittir. Histonlar yüksek oran-da artı yüklü amino asit (Lizin) ve (arjinin) içerir.

Bu aminoasitler eksi yüklü DNA’ya sıkıca bağlanır.

16. Aşağıdaki koşullardan hangisinde hemoglo-binin oksijen doygunluğu diğerlerine kıyasladaha fazladır?

A) pH 7.6

B) 2,3-Difosfogliserat düzeyi çok yüksek

C) Glukoz düzeyi çok yüksek

D) pH 6.0

E) Oksijenin kısmi basıncı düşükÇÖZÜM:

Glioksizom adı verilen özelleşmiş peroksizomlar bitkitohumlarının yağ depolayan dokularında yer alırlar. Buorganel yağ asitlerinin şekere dönüşümünü başlatanenzimler içerir.

Filizlenmekte olan tohum, oluşan bu şekeri enerji kay-nağı olarak kullanır. Fizyolojik O

bağlama eğrisi2

(kPa)0 2 4 6

0 10 30 4020 (Torr)

pO2

1

2

3

4

5

6

00

20

40

60

80

100[HbO ]2 [O ]2 Kanda

mmol/lg/l

DPG pH

CO2 Temp

Sağa kayma

DPG pH

CO2 Temp

Sola kayma

pH7.

6pH

7.4 pH

7.2-7.4

pH7.2

Page 13: Ortaokul - Lise - ALTIN NOKTA · Endoplazmik Retikulumda sentezlenir C) Granüllü Endoplazmik Retikulum yapısında yer alan proteinlerin sentezi GER’da gerçekleşir D) Sitoplazmada

339

2012 - 20. Ulusal Biyoloji Olimpiyatı 1. Aşama Soruları ve Çözümleri

alt

ın n

ok

ta y

ay

ınla

rı ©

ÇÖZÜM:

β oksidasyon uzun zincirli yağ asitlerinin asetilCoA’lara parçalandığı bir süreçtir. Asetil CoA sitrik asitçevrimi sürecine dahil olabilen bir moleküldür.

β oksidasyon aşağıdaki süreçleri kapsar.

1- Açil CoA dehidrogenez ile dehidrogenasyon.Sonuç 1FADH2

2- Enoil CoA hidrataz ile yapılan hidrasyon

3- 3 hidroksiaçil CoA dehidrogenaz ile yapılandehidrogenasyon sonuç 1NADH

4- Tiyolaz ile yapılan kırılma sonuç 1 asetil CoA ve2 karbonu eksiltilmiş yeni bir açil CoA

Bu döngü tüm yağ asidi asetil CoA’lara parçalananadek sürer.

20 karbonlu bir yağ asidinin 10 asetil CoA ya dönüş-mesi için 9 adet β oksidasyon döngüsü gerçekleşir.

17. Yirmi (20) karbonlu bir yağ asidi olanAraşidonikasitin oksidasyonunda kaç adet β-Oksidasyon döngüsü gerçekleşir?

A) 10 B) 9 C) 8 D) 11 E) 12

ÇÖZÜM:

Lösin dallanmış yapılı amino asitlerden birisidir. İnsanvücudunda sentezlenemez ve temel bir amino asittir.Formülü aşağıdaki gibidir.

CH3 CH3

CH

CH2

CH N3 C

HO

OH

19. İnsanda dallanmış yapılı amino asitler esansiyel(temel) olup sentezlenemez ve dışarıdan alın-maları gerekir. Aşağıdakilerden hangisi dal-lanmış yapılı esansiyel amino asitlerdendir?

A) Triptofan B) Lösin C) Asparajin

D) Fenilalanin E) Metiyonin

ÇÖZÜM:

Glikoliz tepkimelerinde glikoz hücreye girer, hekzoki-naz enzimi tarafından fosforile edilir. Hekzokinaz ATPden şekere bir fosfat grubu aktarır. Hekzokinaz trans-ferazlar grubundan bir enzimdir. Hidrolazlar hidrolizreaksiyonlarını katalizler. Ligaz DNA uçlarını birleştirir.İzomerazlar molekülleri izomerlerine çevirir.

Oksidoredüktaz oksidasyon-redüksiyon reaksiyonları-nı katalizler.

18. Aşağıdakilerden hangisi Glukozun Glukoz-6-Fosfat’a dönüşüm reaksiyonunu katalizleyenenzim sınıfıdır?

A) Hidrolaz B) Oksidoredüktaz C) Transferaz

D) Ligaz E) İzomeraz

ÇÖZÜM:

* Amilaz enzimi nişasta ve glikojenin hidrolizinisağlar. Proteinlerin sindirimlerinde rol oynamaz.

* Fosfolipazlar; fosfolipidlere etki etmektedir.Fosfolipazlar ester bağını parçalar. Yani yağlarıhidrolize ederler. Glikojenin sindiriminde rol oyna-mazlar.

* Amino terminal amino asidini tanımlaması içindansil klorür, dabsil klorür, 1-flora-2, 4-dinitro-benzen gibi farklı reaktifler geliştirilmiştir. Aminoterminali reaktif ile işaretlendikten sonra peptit hid-rolizlenir ve işaretli olan aminoasit tanımlanır.Ancak bu yöntemle sekuans tayini yapılamazçünkü hidroliz basamağı peptidi parçalar.

* EDMAN sekansıyla proteinlerin jelde parçalanma-sı sağlanır. Bu kimyasal teknik, proteinlerin N-ter-minal ucunu kullanarak amino asitlerin tek tekeldesine olanak sağlar. Protein hidrolizinde roloynar. Karbonhidrat analizinde rol oynamaz.

* Yağlar lipaz enziminin etkisi ile gliserol ve yağasitlerine hidroliz edilir.

20. Aşağıdaki eşleşmelerden hangisi doğru ola-rak verilmiştir?

A) Amilaz – Protein

B) Dansilklorür - C-terminal analizi

C) Lipaz - Nötral yağ

D) Fosfolipaz – Glikojen

E) EDMAN - Karbohidrat analizi

Page 14: Ortaokul - Lise - ALTIN NOKTA · Endoplazmik Retikulumda sentezlenir C) Granüllü Endoplazmik Retikulum yapısında yer alan proteinlerin sentezi GER’da gerçekleşir D) Sitoplazmada

340

2012 - 20. Ulusal Biyoloji Olimpiyatı 1. Aşama Soruları ve Çözümleri

alt

ın n

ok

ta y

ay

ınla

rı ©

ÇÖZÜM:

Hemoglobinin bir polipeptit zincirini sentezleyen gendebir nokta mutasyon olur. Polipeptit zincirine yanlışlıklavalin amino asidinin girmesiyle alyuvarlar orak ya dahilal şekline dönüşür. Bu anormal kan hücreleri küçükkan damarlarından geçemez.

Bu mutasyon nükleotit değişimi mutasyonu olarakadlandırılır.

23. Orak hücresi anemisi hastalığı ne tip birgenetik değişim sonucunda ortaya çıkar?

A) Nükleotit değişimi mutasyonu

B) Nükleotit delesyonu (eksilmesi) mutasyonu

C) Nükleotit insersiyonu (eklenmesi) mutasyonu

D) Kromozom sayı mutasyonu

E) Kromozom parça kopma mutasyonu

ÇÖZÜM:

TATA kutusu (diğer adıyla Goldberg-Hogness kutusu)arkeler ve ökaryotların genlerinin promotor bölgelerin-de bulunan bir DNA dizisidir. Trankripsiyon faktörlerininve histon proteinlerinin bağlanma bölgesidir.

24. Günümüz bilgileri ışığında; Archaea (Arkeler),Bacteria (Bakteriler) ve Eukarya (Ökaryotlar)domeynleri için aşağıdaki tabloda verilen karak-terler hangi şıkta yanlış olarak verilmiştir?

+ : var/evet - : yok/hayır

21. Bir hücrede RNA molekülü başlıca mesajcı RNA(mRNA), taşıyıcı RNA (tRNA) ve ribozomal RNA(rRNA) olarak bulunur. Bu RNA’ların toplamRNA içerisindeki bulunma oranları sırası,aşağıdaki seçeneklerden hangisinde doğruolarak verilmiştir?

A) mRNA > tRNA > rRNA

B) rRNA > mRNA > tRNA

C) rRNA > tRNA > mRNA

D) mRNA > rRNA > tRNA

E) tRNA > mRNA > rRNA

22. Bildiğiniz gibi, gen ifadesinin translasyon aşa-masında mRNA üzerindeki kodonun 5'→3'yönünde üçüncü bazı, tRNA üzerindeki antiko-donun 5'→3' yönünde birinci bazı ile eş yapmak-tadır. Söz konusu eşleşme noktasında tRNA’da“inozin” mevcut ise kodondaki hangi baz veyabazlarla eş yapabileceği aşağıdaki seçenekler-den hangisinde en doğru şekilde verilmiştir?

A) Yalnız Adenin

B) Adenin veya Guanin

C) Guanin veya Sitozin

D) Adenin, Guanin veya Sitozin

E) Adenin, Urasil veya Sitozin

Özellik Archaea Bacteria Eukarya

A) Histon proteinler + - +

B) Promotor bölgede TATA kutusu - - +

C) Sitoplazmada 70S ribozom boyutu + + -

D) Polisistronik mRNA + + -

E)Başlangıç amino asiti formil-meti-yonin

- + -

ÇÖZÜM:

Bir hücrede en fazla bulunan RNA çeşidi rRNA dır.Sentezlenen tRNA lar hücrede sürekli bulunur. mRNAise sentezlenir, kullanılır ve parçalanır.

ÇÖZÜM:

İnozin, riboz şekerine bir pürin türevi olan hipoksanti-nin bağlanmasıyla oluşan bir nükleozittir. Bu nükleozittRNA’larda yaygın olarak bulunur ve adenin, sitozin veurasil bazlarıyla eşleşebilir.

Solda inozin nükleoziti,aşağıda ise bazı pürinçeşitleri görülmektedir.

O

NH

N

OH OH

CH OH2

O

N

N

purine

theobrmine

adenine

caffeine

guanine

uric acid

hypoxanthine

isoguanine

xanthine

1

6

2

7

3

8

4

9

5

N

N

N

N

N

HN

N

N

N

NH

N

NH

N

NH

NH

NH

NH

NHN

N

N

N

N

NH

N

N

NH

N

HN

N HN

HN

HN

HN

HN

O O

H N2

O O

O

O O

N

O

O

NH2

NH2O O O

Page 15: Ortaokul - Lise - ALTIN NOKTA · Endoplazmik Retikulumda sentezlenir C) Granüllü Endoplazmik Retikulum yapısında yer alan proteinlerin sentezi GER’da gerçekleşir D) Sitoplazmada

341

2012 - 20. Ulusal Biyoloji Olimpiyatı 1. Aşama Soruları ve Çözümleri

alt

ın n

ok

ta y

ay

ınla

rı ©

ÇÖZÜM:

DNA(Kod)

mRNA(Kodon)

tRNA(Antikodon)

T A UC G CA U A

25. DNA’daki bir kodonun nükleotit dizisinin TCAolduğunu farzedin. Bu üçlünün kalıp olarak kul-lanılmasıyla mRNA’da elde edilen kodonla eşyapacak tRNA üzerindeki antikodon dizisi aşağı-dakilerden hangisi olacaktır?

A) AGU B) UCA C) TCA D) UCU E) AUC

ÇÖZÜM:

Transpeptidaz enzimi peptigolikan zincirleri arasındabağlantı kurarak sağlam bir duvar oluşturulmasını sağ-lar. Penisilin antibiyotiğin transpeptidaz enzimine bağ-lanarak bu enzimi işlevsiz hale getirir. Bu sayede hücreduvarı sentezi durdurulmuş olur.

26. Bakteri hücre duvar yapısında bulunan pepti-doglikan için aşağıdaki ifadelerden hangisiyanlıştır?

A) Lizozim peptidoglikandaki β(1,4)-glikozit bağlarınıparçalar

B) Peptidoglikanda bazı amino asitlerin D-formlarıbulunur

C) Peptidoglikan Gram pozitif bakterilerde hücreduvarının büyük bir bölümünü oluşturur

D) Penisilin peptidoglikandaki transpeptit bağlarınıparçalar

E) Peptidoglikan Gram negatif bakterilerin hücreduvarında Gram pozitiflere göre daha az orandabulunur

ÇÖZÜM:

Kemoorganotrof; kimyasal bileşikleri enerji, organikmaddeleri elektron kaynağı olarak kullanan organiz-madır.

* Kemoorganotrofi Bu canlıların gerçekleştirdiğiolaylardır.

* Kemolitotrof kimyasal bileşikleri enerji, inorganikmaddeleri elektron kaynağı olarak kullanan orga-nizmadır.

* Kemolitotrofi ise bu canlıların gerçekleştirdiğiolaydır.

Fototrof ışığı enerji, karbondioksiti karbon kayna-ğı olarak kullanan organizmadır.

* Fototrofi ise bu canlıların gerçekleştirdiği olaydır.

28. Kemoorganotrofi, Kemolitotrofi ve Fototrofiiçin aşağıda verilen tanımlamalardan hangisiyanlıştır?

A) Kemoorganotrofide biyosentez için organik bile-şikler kullanılır

B) Kemolitotrofide terminal (son) elektron akseptörle-ri (tutucuları) inorganik maddelerdir

C) Fototrofide biyosentez için CO2 veya organik bile-şikler kullanılır

D) Kemolitotrofide biyosentez için CO2 kullanılır

E) Kemoorganotrofide biyosentez için CO2 kullanılır

ÇÖZÜM:

Glikozun sitoplazmada oksijen kullanılmadan iki pirü-vik asite parçalanması tüm canlıların tüm hücrelerindegerçekleşen ortak bir olaydır. Bu olayda kullanılanenzimler tüm canlılarda ortaktır.

Pirüvik asitten sonraki reaksiyonlarda farklı enzimlerkullanıldığından farklı yan ürünler oluşur. Canlıda bulu-nan enzimin çeşidine göre pirüvik asitten; etil alkol,laktik asit, asetik asit gibi son ürünler oluşur.

27. Fermentasyonda farklı son ürünlerin oluşma-sında kullanılan anahtar ara molekül aşağı-dakilerden hangisidir?

A) Laktik asit B) Piruvik asit C) Asetik asit

D) Formik asit E) Sitrik asit

Page 16: Ortaokul - Lise - ALTIN NOKTA · Endoplazmik Retikulumda sentezlenir C) Granüllü Endoplazmik Retikulum yapısında yer alan proteinlerin sentezi GER’da gerçekleşir D) Sitoplazmada

342

2012 - 20. Ulusal Biyoloji Olimpiyatı 1. Aşama Soruları ve Çözümleri

alt

ın n

ok

ta y

ay

ınla

rı ©

ÇÖZÜM:

Bir hücrenin çekirdeğinde bulunan kromozom DNA nınproteinle kaplanmış halidir. DNA üzerinde genler bulu-nur. Genler üçlü şifrelerden oluşur.

Her bir üçlü şifrede üç nükleotit bulunur. Bir nükleotit;bir azotlu organik baz, bir deoksiriboz şeker ve bir fos-fattan oluşur. Azotlu bazın şekere bağlanmasıyla olu-şan moleküle de nükleozit denir.

29. Aşağıdaki seçeneklerin hangisinde yapılaren büyükten en küçüğe doğru olarak sıralan-mıştır?

A) Gen > Kromozom > Kodon > Nükleozit > Nükleotit

B) Gen > Kodon > Kromozom > Nükleotit > Nükleozit

C) Kromozom > Gen > Kodon > Nükleotit > Nükleozit

D) Kodon > Kromozom > Gen > Nükleozit > Nükleotit

E) Kromozom > Kodon > Gen > Nükleotit > Nükleozit

ÇÖZÜM:

Diploit hücrelerden gamet oluşması için mayoz bölün-me olması gerekir. Somatik hücrelerin çoğalması,yaraların iyileşmesi, büyüme ve aynı genotipli yenihücrelerin oluşması mitozla gerçekleşir.

31. Aşağıdakilerden hangisi insanda mitozunişlevleri arasında sayılmaz?

A) Somatik hücrelerin çoğalması

B) Yaranın iyileşmesi

C) Büyüme

D) Aynı genotipli yeni hücrelerin oluşması

E) Diploit hücrelerden gamet oluşumu

ÇÖZÜM:

Hücre döngüsü interfaz ve bölünme evresi olmaküzere iki bölümden oluşur. İnterfaz G1 S ve G2 de RNAsentezi, ATP sentezi ve protein sentezi olur. S evresin-de fazladan birde DNA sentezi olacağından G1 ve G2evresi arasında DNA miktarı iki katına çıkar.

32. Hücre döngüsünün hangi evresinde DNAmiktarı 2 katına çıkar?

A) Hücre döngüsünün G1 → G2 fazları arasında

B) Mitozun profaz → anafaz evreleri arasında

C) Mitozun metafaz → anafaz evreleri arasında

D) Mayozun metafaz → anafaz evreleri arasında

E) Mayozun profaz → anafaz evreleri arasında

ÇÖZÜM:

Kloroplast, kromoplast, lökoplast ve amiloplastlar plas-titlerdir. Protoplastit denilen ön plastitlerden oluşurlar.Bütün plastitler birbirine dönüşebilirler.

Protoplast hücre duvarı olmayan sitoplazma kütlesidir.

33. Aşağıdaki organellerden hangisi proplastid-lerden başkalaşmaz?

A) Kloroplast B) Kromoplast C) Protoplast

D) Lökoplast E) Amiloplast

ÇÖZÜM:

Apikal meristemin tam üstünde apikal meristeminbölünmesiyle oluşan hücreler iç içe geçmiş silindirlerşeklinde üç hücre tabakası oluşturur. Bu hücreler belir-li bir süre bölünmeyi sürdürür.

Bunlar, protoderm, prokambiyum ve temel meristemdenen pirimer meristemlerdir. Bu pirimer meristemlerkökün üç pirimer doku sistemini yani örtü doku iletimdokusu ve temel dokuyu üretir.

34. Aşağıdakilerden hangisi birincil meristemdir?

A) Protoderm B) Mantar Kambiyumu

C) Kolenşima D) Lentisel

E) Kök tüyleri

ÇÖZÜM:

En son bilgilerimize göre bir sperm hücresinde yakla-şık 20-25 bin gen bulunduğu tahmin edilmektedir.

30. En son bilgilerinize göre bir insan sperm hüc-resinde yaklaşık kaç adet gen mevcuttur?

A) Yaklaşık 1.000 B) Yaklaşık 20.000

C) Yaklaşık 100.000 D) 23

E) 46

Page 17: Ortaokul - Lise - ALTIN NOKTA · Endoplazmik Retikulumda sentezlenir C) Granüllü Endoplazmik Retikulum yapısında yer alan proteinlerin sentezi GER’da gerçekleşir D) Sitoplazmada

343

2012 - 20. Ulusal Biyoloji Olimpiyatı 1. Aşama Soruları ve Çözümleri

alt

ın n

ok

ta y

ay

ınla

rı ©

ÇÖZÜM:

Kök şapkası çok narin olan meristemi fiziksel olarakkorur. Ayrıca kök şapkası büyümekte olan kök ucuçevresindeki toprağı gevşeten kaygan bir polisakkaritsalgılar. Boyca uzama kök ucunun yakınında gerçekle-şir. Kök şapkası yerçekimini algılayarak kökün pozitifjeotropizma göstermesini sağlar.

35. Kökte yer çekimi nerede algılanır?

A) Kök tüylerinde B) Genişleme bölgesinde

C) Uç meristemde D) Kök şapkasında

E) Olgunlaşma bölgesinde

ÇÖZÜM:

Partenokarpı tohum taslağının doğal veya yapay yol-larla döllenme olmasızın meyve üretilmesi olayıdır.Dolayısıyla bu meyveler çekirdeksizdir. Yani çekirdek-siz meyveler partenokarpiktir.

36. Döllenme olmadan oluşan çekirdeksiz mey-veler için aşağıdaki tümcelerden hangisidoğrudur?

A) Bunlar apomiktiktir

B) Absisik asit uygulaması ile oluşurlar

C) Bunlar partenokarpiktir

D) Bunlarda mezokarp olmaz

E) Sadece gülgillerden elde edilirler

37. Köklere özel olup gövdede bulunmayandoku hangisidir?

A) Parankim B) Endoderm C) Korteks

D) Epiderm E) Floem

ÇÖZÜM:

Bazı kara bitkilerinde bulunan endoderm korteksin eniç kısmındaki hücre tabakasıdır. Suyun apoplast yolun-dan girişini engelleyen kaspari şeridini bulundurur. Su,iyon ve hormonların vasküler sisteme giriş-çıkışınıdenetler. Gövdede bulunmaz, köke özgü bir yapıdır.

ÇÖZÜM:

Sekonder vasküler dokusunun yıllar boyunca birimi birodunsu bitkideki çap artışının büyük bir bölümündensorumludur.

Vasküler kambiyum içe doğru sekonder ksilemi dışadoğru sekonder floemi oluşturur. Vasküler kambiyu-mun oluşturduğu silindirin çapı zamanla arttıkça ağaçenine büyür. Sekonder dokuların ardışık tabakaları üstüste gelir. Her tabakanın çapı bir öncekinden dahageniştir.

Protoderm, prokambiyum ve temel meristem örtü dokuiletim dokusu ve temel dokuyu oluşturur. Periderm piri-mer bitki gövdesinin epidermisinin yerini alan sekon-der bitki gövdesinin koruyucu kabuğudur.

38. Bir ağacın eninin artmasından aşağıdakiler-den hangisi sorumludur?

A) Prokambiyum B) Temel meristem

C) Protoderm D) Periderm

E) Vasküler kambiyum

ÇÖZÜM:

Gutasyon çimenlerin yaprak ayalarının uç kısımlarındaya da sabahları bazı otsu dikotillerin yaprak kenarların-dan su damlalarının dışarı atılmasıdır. Kök basıncı ksi-lem özsuyunu gövde sistemine iter. Bunun sonucuyapraklara transpirasyonla kaybedilenden daha fazlasu girer ve fazla su gutasyon sıvısı olarak dışarı itilir.

39. Gutasyonda sıvı olarak su kaybedilmesinisağlayan nedir?

A) Kök basıncı B) Transpirasyon (terleme)

C) Ozmoz D) Fotosentez

E) Stomalar

ÇÖZÜM:

Birçok bitkide, kaybedilen suyun fotosentezle kullanı-lan her 1 gr. CO2 ye oranı 600:1 dir. Bu karbonhidratadönüştürülen her gram CO2 ye karşılık 600 gr. suyunkaybedildiği anlamına gelir. Bu oran C4 bitkileri için300:1 dir. Sonuçta bitkiler aldıkları suyun % 90 danfazlasını transpirasyonla kaybederler.

40. Çiçekli bir bitkinin aldığı suyun yaklaşık nekadarı terleme (transpirasyon) yoluyla atılır?

A) %90 yada daha fazlası B) %25 yada daha azı

C) Yaklaşık % 75 D) Yaklaşık % 60

E) Yaklaşık % 50

Yanıt: E

Page 18: Ortaokul - Lise - ALTIN NOKTA · Endoplazmik Retikulumda sentezlenir C) Granüllü Endoplazmik Retikulum yapısında yer alan proteinlerin sentezi GER’da gerçekleşir D) Sitoplazmada

344

2012 - 20. Ulusal Biyoloji Olimpiyatı 1. Aşama Soruları ve Çözümleri

alt

ın n

ok

ta y

ay

ınla

rı ©

ÇÖZÜM:

Fotosistem I’den çıkan elektronlar ferrodoksin üzerin-den NADP ye aktarılır. Fotosistem II’den gelen elek-tronlar plastokinon, sitokromlar ve plastosyanin üzerin-den fotosistem I’e aktırılır. Fotosistem I’in elektronaçığı fotosistem II tarafından kapatılır, fotosistem II’ninelektron açığıda su tarafından kapatılır.

41. Fotosentezde, Fotosistem II’den gelen elek-tron Fotosistem I’e geçmeden önce en sonhangi elektron taşıma zinciri elemanındangeçer?

A) Plastokinin B) Plastosiyanin C) Plastomyelin

D) Ferrodoksin E) Ferrokromatin

ÇÖZÜM:

C3 bitkilerinde CO2 ile ilk tepkimeye giren bileşik ribu-loz 1,5 bifosfattır. Ribuloz 1,5 bifosfat CO2 ile birleştik-ten sonra altı karbonlu iki fosfatlı bir bileşik olan karar-sız ara bileşiği oluşturur. Diğer moleküller solunumreaksiyonları ile ilgilidir.

42. C3 bitkilerinde CO2 ile reaksiyona giren ilkmetabolit hangisidir?

A) Oksaloasetat B) Malat C) Süksinat

D) Ribuloz 1,5-bisfosfat E) Piruvat

ÇÖZÜM:

Sitokininler yaprakların geç dökülmesinde etkilidir.

Etilen meyve olgunlaşmasını sağlar.

Absisik asit tohum çimlenmesini engeller.

Oksin hem kök hem gövde büyümesinde etkilidir.

Giberellin gövde uzamasında diğerlerinden daha fazlaetkilidir.

43. Aşağıdakilerden hangisi köke nazaran göv-deyi daha fazla etkiler?

A) Sitokinin B) Etilen C) Absisik Asit

D) Gibberellin E) Oksin

ÇÖZÜM:

Oksin gövde ucundaki meristem dokudan salgılanır vebüyümeyi sağlar. Işık gelen kısımdan karanlık tarafadoğru kaçar. Işık almayan kısım daha fazla büyüdü-ğünden ışığa yönelmeyi de sağlar.

44. Işığın oksin üzerindeki etkisi aşağıdakilerdenhangisidir?

A) Işık oksini daha karmaşık bir kimyasal yapıya dön-üştürür

B) Işık oksini tamamen parçalar

C) Işıktan karanlık tarafa doğru kaçmasına neden olur

D) Sentezini artırır

E) Etkinliğini arttırır

45. Karayosunları, ciğerotları ve boynuzotlarınınortak yönü aşağıdakilerden hangisidir?

A) Stomaları vardır

B) Meristemleri bulunur

C) Farklı cinsiyette bitkiler oluştururlar

D) Spor oluştururlar

E) Gemma üretimi yoluyla eşeysiz ürerler

ÇÖZÜM:

Karayosunları cigerotları ve boynuz otları çiçeksiz bit-kilerdir. Tohum oluşturamazlar spor denilen hücreleroluştururlar. Sporların çimlenmesiyle gametofitler,gametofitlerin oluşturduğu gametlerin döllenmesiyledesporofitler oluşur. Sporofitler tekrar sporları oluşturur.

ÇÖZÜM:

Petal; taç yapraktır göz alıcı ve parlak renklerdedir.Fotosentez yapmaz.

Stigma; tepeciktir. Türe göre farklı renklerde olabilir.

Stamen; erkek organlardır. Yeşil değildir.

Karpel; yumurtalıktır. Türe göre farklı renklerde olur.

Sepal; çanak yapraklardır. Yeşildir ve fotosentezyapar.

46. Aşağıdaki çiçek bölümlerinden hangisi yeşilve yaprağımsıdır?

A) Petaller B) Stigmalar C) Stamenler

D) Karpeller E) Sepaller

Page 19: Ortaokul - Lise - ALTIN NOKTA · Endoplazmik Retikulumda sentezlenir C) Granüllü Endoplazmik Retikulum yapısında yer alan proteinlerin sentezi GER’da gerçekleşir D) Sitoplazmada

345

2012 - 20. Ulusal Biyoloji Olimpiyatı 1. Aşama Soruları ve Çözümleri

alt

ın n

ok

ta y

ay

ınla

rı ©

ÇÖZÜM:

Bir evcikli monoik bitkide erkek ve dişi çiçekler aynıbitki üzerinde bulunur. Bitki kendi kendini de dölleyebi-lir, çapraz tozlaşma da yapabilir.

47. Bir evcikli (monoik) bir bitki:

A) Aynı çiçek üzerinde hem erkek hem dişi çiçek bö-lümlerine sahiptir

B) Aynı bitkide ayrı ayrı erkek ve dişi çiçeklere sahiptir

C) Erkek çiçek bir bitkide, dişi çiçek farklı bir bitkidedir

D) Sadece kendine döllenmektedir

E) Sadece dışa döllenmektedir

ÇÖZÜM:

Kupfer hücreleri karaciğerde bulunan ve retikula endo-talyal sistemin bir bölümünü oluşturan özelleşmiş mak-rofajlardır. Fagositoz yaparlar, safra salgısı, asit salgı-sı, mukus salgısı ve sinir iletimi yapmazlar.

48. Kuppfer hücrelerinin görevi aşağıdakilerdenhangisidir?

A) Fagositoz B) Safra salgısı C) Asit salgısı

D) Mukus salgısı E) Sinir iletimi

ÇÖZÜM:

Glomerulus kılcalları vücuttaki diğer kılcallardan farklıolarak iki atardamar arasında olduğundan sabit veyüksek kan basıncına sahiptir. Bu basınç glomerulus-tan bowman kapsülüne yalnızca süzülmenin olmasınaizin vermektedir. Henle kulpunun farklı bölgelerindeince yapılı, özellikleri farklı olan dört tip yassı epitelhücre bulunur. İnen kol tek katlı yassı epitel hücreler-den oluşur. Bu hücreler suya geçirgen olup tuzlarageçirgen değildir.

Çıkan kol tek katlı kübik epitel hücrelerinden oluşur. Bukısım suya geçirgen değildir, tuzun emilimini sağlar.Distal tüp ve toplama kanalının korteks kısmı üreyegeçirgen değil suya geçirgendir. Salgılama (aktif pom-palama) işlemi henle kulpunun çıkıcı ince kolundadeğil distal tüpte gerçekleşir.

49. Nefronun bölümleri ile ilgili aşağıdaki ifade-lerden hangisi yanlıştır?

A) Glomerulus kılcallarında geri emilim yoktur

B) Henle kulpu inici kolu suya geçirgen, tuzlara geçir-gen değildir

C) Henle kulpu çıkıcı ince kolunda tuz aktif olarakmedullaya pompalanır

D) Henle kulpu çıkıcı kalın kolu suya geçirgen değildir

E) Toplayıcı kanalların kortekste kalan kısmı üreyegeçirgen değildir

ÇÖZÜM:

Medulla oblangata beyin ve omurilik arasında yer alır,beyin ve omurilik arasındaki bağlantıyı sağlar. Buradabeyinin sağ lobundan gelen sinirler vücudun sol tarafı-na; sol lobundan gelen sinirler beyinin sağ tarafınaçapraz geçer.

Ayrıca solunum, sindirim dolaşım gibi yaşamsal olay-ları düzenler. Hücresel solunum sonucu artan CO2kanın asitliğini artırdığında etkilenir ve solunumu hız-landırır.

50. Medulla oblongata aşağıdakilerden hangisin-den etkilenir ve düzenler?

A) Kanın pH seviyesi

B) Vücut sıcaklığı

C) Kanın oksijen miktarı

D) Kandaki laktik asit miktarı

E) Akciğerlerdeki hava miktarı

ÇÖZÜM:

Midede ezilmiş ve bulamaç haline getirilmiş besinlerekimus denir. Kimusun duedonuma geçmesiyle duedo-num hücreleri asidik kimustan etkilenerek Sekretinhormonunu salgılar. Sekretin pankreası uyararakHCO3

− iyonlarının salgılanmasını uyarır. Kanın şekeri-nin azalması glukagon, herhangi bir yaralanma hista-min, mideye besin gelmesi gastrin, duedonuma besingelmesi kolesistokinin salgılanmasını uyarır.

51. Asidik kimusa cevap olarak duodenumdan sal-gılanan hormon aşağıdakilerden hangisidir?

A) Gastrin B) Kolesistokinin C) Sekretin

D) Glukagon E) Histamin

52. Kalp aktivitesinin düzenlenmesi ile ilgili aşa-ğıdakilerden hangisi yanlıştır?

A) Kalbin ‘‘intrinzik düzenlemesi”, kalbin kendi içindeolan mekanizmaları ifade eder

B) Artan ‘‘önyük”, ventriküllerin gerilimini arttırarakatım hacminin artmasına yol açar

C) Kardiyak verim, atım hacminin artması ile artar

D) Kardiyak verim, atım sayısının artması ile artar

E) Kardiyoregülatör merkez beynin motor korteksin-de yer alır

Page 20: Ortaokul - Lise - ALTIN NOKTA · Endoplazmik Retikulumda sentezlenir C) Granüllü Endoplazmik Retikulum yapısında yer alan proteinlerin sentezi GER’da gerçekleşir D) Sitoplazmada

346

2012 - 20. Ulusal Biyoloji Olimpiyatı 1. Aşama Soruları ve Çözümleri

alt

ın n

ok

ta y

ay

ınla

rı ©

ÇÖZÜM:

- “intrinzik düzenleme” iç kaynaklı, ilgili organınkendisiyle ilgili faktörlerle düzenleme yapmasıanlamına gelir.

- Kalbin “intrinzik düzenlemesi” kalbin kendi içindeolan düzenlemeyi ifade eder.

- Önyük: Diastol sırasında karıncığı dolduran,kan hacmi anlamına gelmektedir. Artan önyükventriküllerin hacmini artıracağından basınçartar ve aorta giden damardada atım hacmindeartış gözlenir.

- Diastol başlangıcında, diastolik kan basıncı vekoroner kan akımı artar. Buna bağlı olarak kardi-yak verim artışı gözlenir.

- Kardiyoregülatör merkez beyin sapında bulu-nur ve kalbin çalışmasını düzenler.

53. Sistemik arteriyel karbondioksit kısmi basıncınınartışına karşılık solunum sisteminin cevabındarol oynayan en önemli afferent (duyu) resep-törler aşağıdakilerden hangisidir?

A) Aortik ve karotid cisimlerin CO2 reseptörleri

B) Aortik ve karotid cisimlerin H+ reseptörleri

C) Beynin medullasındaki CO2 reseptörleri

D) Beynin medullasındaki H+ reseptörleri

E) Akciğer ve solunum yollarındaki CO2 reseptörleri

ÇÖZÜM:

Medulla oblangata arteriyel kan pH1, değişimlerineoldukça duyarlıdır ve solunum merkezlerinin diğerbölümlerini uyarır. Kan beyin bariyerinden H geçemezCO2 geçer. Kanda PCO2 arttığı zaman hem interstiyelsıvıda hem de BOS da PCO2 miktarı artar.

CO2 bu sıvılarda su ile reaksiyona girer ve H+ iyonlarımeydana gelir. Kanda CO2 arttığı zamankinden dahafazla H+ serbest kalır. BOS daki H+ iyonları medularreseptörleri uyarmak üzere dokuya geçer. En önemlireseptör medulladaki H+ reseptörleridir.

ÇÖZÜM:

- Midede asit üreten hücreler “Parietal” hücreler-dir. Bu hücrelerin asit salgılamasını kontrol eden“G hücreleridir”. G hücreleri gastrin hormonuüretir. Bu hormon parietal hücrelerin HCI salgıla-masını uyarır.

- Pepsinojenler oxyntikbez alanında (ve daha azoranda pilarik bez bölgesinde) şef hücreler tara-fından sentezlenir.

- İntrinzik faktör midenin mukoz membranındakiparietal hücrelerde HCI ile birlikte üretilmektedir.

- Pankreatik asinar hücreler sindirim enzimlerininsalgılanmasını düzenler. Proelastaz tripsin ileelastoza dönüşür. Nötral alifatik zincirli amino-asitler arasındaki peptit bağlarını parçalar.

- Enterokromoffin benzeri hücreler; vücuttaserotonin ve protoglandinler gibi bir kısım otoko-itleri üreterek depolayan mide-bağırsak mukoza-sı tipidir. Enterokinaz enterokromoffin benzerihücrelerin salgısı değildir. On iki parmak bağır-sağındaki brünner salgı bezlerinden salgılanır.

54. Sindirim sisteminde yer alan hücreler ve salgıla-rı aşağıdakilerden hangisinde yanlış eşleşti-rilmiştir?

A) G hücreleri – Gastrin

B) Paryetal hücreler – İntrinzik faktör

C) Şef hücreler – Pepsinojen

D) Pankreatik asinar hücreler – Proelastaz

E) Enterokromaffin benzeri hücreler – Enterokinaz

55. İdrarda çok miktarda amonyum iyonlarının(NH4

+) görülmesi aşağıdaki durumlardan han-gisine veya hangilerine verilen renal (böbrek)cevaptır?

I. Solunuma bağlı asidoz

II. Metabolizmaya bağlı asidoz

III. Solunuma bağlı alkaloz

IV. Metabolizmaya bağlı alkaloz

V. Diabetes mellitusa (şeker hastalığı) bağlı asidoz

A) Yalnız I B) Yalnız II C) I ve II

D) III ve IV E) I, II ve V

Yanıt: E

Page 21: Ortaokul - Lise - ALTIN NOKTA · Endoplazmik Retikulumda sentezlenir C) Granüllü Endoplazmik Retikulum yapısında yer alan proteinlerin sentezi GER’da gerçekleşir D) Sitoplazmada

59. Aşağıdaki şekilde Nöron A ve Nöron B’de oluşanAksiyon Potansiyelleri (AP), akson ucuna ulaştı-ğı ve nörotransmitterlerini serbest bıraktıklarıhalde, Nöron C’de AP oluşmamaktadır.

Buna göre aşağıdaki seçeneklerden hangisiyanlıştır?

A) Nöron A uyarıcı nörotransmitter salgılarken, NöronB inhibe edici nörotransmitter salgılamaktadır

B) Nöron A’daki Aksiyon potansiyeli akson ucuna ulaş-tığı ve nörotransmitterini serbest bıraktığı zaman,Nöron C’nin dinlenme membran potansiyelinde birdeğişiklik olacaktır

C) Nöron A ve Nöron B’den gelen sinyaller, Nöron C’yieşik değere ulaştıramamıştır

D) Nöron C mutlak refrakter dönemde olabilir

E) Nöron A’daki Aksiyon Potansiyeli Nöron C’ye ulaş-madan önce daha küçük dört Aksiyon Potansiyelineayrılmıştır

347

2012 - 20. Ulusal Biyoloji Olimpiyatı 1. Aşama Soruları ve Çözümleri

alt

ın n

ok

ta y

ay

ınla

rı ©

ÇÖZÜM:

Solunuma bağlı asidoz kanda PCO2 yükselmesidir.(PCO2 > 50)

Metabolik asidoz; metabolik veya diğer sebeplerleveya aşırı bikarbonat kaybına bağlı artmış hidrojenüretimi sonucu oluşur.

Diabetes mellitusa bağlı asidoz=şeker yüksekliğineve kullanılmamasına bağlı kanda asit artışıdır.

Alkaloz ise kanda ve diğer vücut sıvılarında alkali faz-lası olmasıdır. Normalde kanın pH’ı 7,35 - 7,45 arasın-da tutulur. Ancak alkaloz durumda pH bu düzeyin üze-rine çıkar. İdrarda çok miktarda amonyum (NH4) iyonugörülmesi asidoz durumda gözlenen renal (böbrek)cevaptır.

ÇÖZÜM:

Toplam akciğer kapasitesi akciğerlerin mümkün olanen büyük inspirasyon hareketi sırasında akciğerlerdebulunan maksimum hava miktarıdır.

Vital kapasite + reziduel hacim = (akciğerde kalan hava)

= Total akciğer kapasitesi

değeri 4600+1200 = 5800 ml dir.

56. Aşağıdakilerden hangisi toplam akciğer kap-asitesini gösterir?

A) Anatomik ölü boşluk + Alveolar ölü boşluk

B) Rezidüel hacim + Vital kapasite

C) Vital kapasite + Tidal hacim

D) Rezidüel hacim + Tidal hacim

E) Fonksiyonel rezidüel kapasite + Ekspirasyonyedek hacmi

ÇÖZÜM:

Parasempatik sinir sisteminin devreye girmesiyle kalpatım hızı solunum hızı, pupil çapı ve solunum yolları-nın çapı azalır. Sindirim sisteminin organlarının çalış-ması hızlanır.

Sempatik sinir sistemi devreye girdiğinde ise sindirimsisteminin çalışması azalırken diğer durumlarda artmagözlenir.

57. Parasimpatik sinir sistemi aşağıdakilerdenhangisini artırıcı yönde etkiler?

A) Kalp hızı B) Solunum hızı C) Sindirim

D) Pupil çapı E) Solunum yollarının çapı

ÇÖZÜM:

Hipotalamus salgıladığı faktörlerle hipofizi etkiler, hipo-fiz de endokrin sistemi kontrol eder.

Yeme, içme, uyku, uyanıklık, açlık, tokluk, kilo, iştah,hiddet, kızgınlık, üreme, gelişme, seksüel davranışlar,vücut ısısı, kanın su dengesi gibi olayları hipotalamusdüzenlerken, solunum, dolaşım, sindirim gibi olaylarıomurilik soğanı düzenler.

58. Aşağıdakilerden hangisi hipotalamusunfonksiyonlarından biri değildir?

A) Hipofiz bezinin endokrin aktivitesini düzenlemek

B) İştahı düzenlemek

C) Solunumu düzenlemek

D) Vücut sıcaklığını düzenlemek

E) Sirkadiyen ritimleri düzenlemek

A

B

C

Bir APBir AP

Page 22: Ortaokul - Lise - ALTIN NOKTA · Endoplazmik Retikulumda sentezlenir C) Granüllü Endoplazmik Retikulum yapısında yer alan proteinlerin sentezi GER’da gerçekleşir D) Sitoplazmada

348

2012 - 20. Ulusal Biyoloji Olimpiyatı 1. Aşama Soruları ve Çözümleri

alt

ın n

ok

ta y

ay

ınla

rı ©

ÇÖZÜM:

Aksiyon potansiyeli akson üzerinde akarken büyüklükbakımından sabit kalır. Nöron A ve nöron B de oluşanaksiyon potansiyelleri akson ucuna ulaştığı haldenöron C de aksiyon potansiyeli oluşmamasının nedeniolarak, noron B nin inhibe edici madde salgılaması,nöron A ve nöron B den gelen uyartıların nöron C yieşik değere ulaştıramaması, nöron C nin mutlak ref-rakter dönemde olması gösterilebilir.

Nöron A daki aksiyon potansiyeli nöron C ye ulaşma-dan önce dört küçük aksiyon potansiyeline ayrılma-mıştır. Sadece akson ucuna ulaşmıştır.

60. İnsan ovaryumuna ait aşağıdaki Şekilde verilenrakamların ifade ettikleri yapılar Tabloda veril-miştir.

Şekil ve Tablo dikkate alındığında aşağıdakiifadelerden hangisi yanlıştır?

A) 1, 2, 3, 4 ve 7 gelişmekte olan yumurta hücresinibulundurur

B) 5 ve 6 dejenere olan (bozulan) folikülleri göster-mektedir

C) 9 ovaryuma ait bir yapı değildir

D) 1, 2, 4, 5, 6 ve 7 ovaryumun korteks tabakasındabulunur

E) 5, 8 ve 9 ovaryumun hormon üreten yapılarıdır

7

1

2

3

45

6

9

8

No Yapı

1 Primordial folikül

2 Primer folikül

3 Teka

4 Antrum (sekonder folikül)

5 Corpus albicans (beyaz cisim)

6 Corpus luteum (sarı cisim)

7 Graafian folikül (olgun folikül)

8 Oosit

9 Fimbria

ÇÖZÜM:

Şekildeki 1, 2, 3, 4 ve 7 gelişmekte olan yumurta hüc-resini bulundurur.

5 ve 6 dejenere olan folikül olan korpus luteumdur.

9, fallopi tüpünün ucundaki kirpikli hunidir.

1, 2, 3, 4, 5, 6, 7 nolu yapılar ovaryumun korteks taba-kasında bulunur.

8 nolu yapı II. oosittir, hormon üretmez, 9 nolu yapı fal-lopi tüpüdür, hormon üretmez.

ÇÖZÜM:

Duyu organından aldığı uyartıyı merkezlere taşıyannöronlara duyu nöronu denir. Duyu nöronunun birucu perifere bir ucu merkeze uzanır. Hücre gövdesiperiferde bulunur.

Duyu organlarından gelen duyu sinirleri omurilikle arkakök düğümleriyle bağlantı kurarlar. Duyu nöronlarımutlaka bir reseptörle sinaps yapmak zorunda değildir.Çünkü serbest sinir uçları da duyuları alabilir.

61. Periferden gelen bilgiyi omuriliğe taşıyan birduyu nöronu ile ilgili aşağıda verilen bilgiler-den hangisi yanlıştır?

A) Duyu nöronu mutlaka özelleşmiş bir reseptörhücre ile sinaps yapar

B) Duyu nöronu gangliyonda başka bir nöron ilesinaps yapmaz

C) Duyu nöronunun biri perifere, diğeri merkezi sinirsistemine uzanan iki uzantısı mevcuttur

D) Duyu nöronunun hücre gövdesi merkezi sinir sis-teminin dışındadır

E) Duyu nöronunun hücre gövdesi dorsal kök gangli-yonda bulunur

Page 23: Ortaokul - Lise - ALTIN NOKTA · Endoplazmik Retikulumda sentezlenir C) Granüllü Endoplazmik Retikulum yapısında yer alan proteinlerin sentezi GER’da gerçekleşir D) Sitoplazmada

349

2012 - 20. Ulusal Biyoloji Olimpiyatı 1. Aşama Soruları ve Çözümleri

alt

ın n

ok

ta y

ay

ınla

rı ©ÇÖZÜM:

Şekilde gösterilen kısımlardan

1- Vitroz hümor sıvı (camsı cisim)

2- Kornea

3- Aköz hümor sıvı

4- Sarı benek

5- Retina

6- Damar tabakası (koroid)

7- Siliyer cisimdir.

62. İnsan gözüne ait aşağıdaki şematik şekilderakamların ifade ettikleri yapılar tablodaki seçe-neklerin hangisinde doğru olarak verilmiştir?

İris

Göz bebeğiLens

Optik sinir

4

5

6

7

12

3

Vitröz hümor(sıvı)

KorneaAköz

hümor(sıvı)

Fovea(Sarı nokta) Retina Koroid

Siliyercisim

A) 3 2 1 5 4 6 7

B) 1 2 3 6 5 4 7

C) 2 3 1 4 7 5 6

D) 1 2 3 4 5 6 7

E) 7 3 1 5 4 6 7

ÇÖZÜM:

Eşey kromozomlarında ayrılamama sonucu turnersendromu ortaya çıkar. Gonozom taşımayan bir gametile sadece x gonozomu taşıyan bir gametin birleşme-siyle oluşur.

63. Turner sendromu hastalığı aşağıdakilerden han-gisinin sonucunda ortaya çıkar?

A) Kromozom fizyonu

B) Krosing over

C) Eşey kromozomunun ayrılmaması

D) Kromozom füzyonu

E) İnversiyon

ÇÖZÜM:

Multifaktöryel kalıtım (çok genli kalıtım): Süreklideğişebilen, ölçülebilen özellikleri içeren, Mendeliyenkalıtım tipi göstermeyen kalıtımdır. Toplumdaki dağı-lımları Gauss eğrisine uyar, bunlar sürekli (continuous)özelliklerdir ve en az iki gen çifti tarafından kalıtılırlar.İnsanlara ait bir çok normal özellik poligenik kalıtımgösterir. Bunlar, boy uzunluğu, saç rengi, şekli; deri vegöz rengi, vücut şekli, zeka, vücut indeksi, kan basın-cı, parmak izi, eritrosit büyüklüğü vs. dir.

Pleiotropi: Bir gen birden fazla karaktere etki yapabi-liyorsa bu olaya peliotropi denir. Gen ise peliotropikgendir. Örneğin: Bir gen hem boy uzunluğunca hem decanlının göz rengini kontrol ediyor olabilir.

Epistaz: Birden fazla genin aynı genel fenotipik karak-terleri kontrol ettiği durumlarda görülür. Aynı özelliküzerine farklı şekilde etki eden fakat allel olmayan fark-lı baskın genlerden birinin fenotipte kendini gösterme-sine ve diğer baskın genin etkisini örtmesine epistasidenir. Örneğin: Körlük geni ve mavi göz rengi birarayageldiğinde körlük geni dominantsa göz rengi ortayaçıkmayabilir.

Eşbaskınlık: Genetikte çaprazlanan homozigot ikigenin dölünün genotip olarak heterozigot olmasınarağmen bir karakterin diğerine baskın çıkmaması veikisinin de eşit derecede fenotipte kendini göstermesidurumudur. Örneğin: A B 0 kan grubu sistemi.

Bağlantılı gen kalıtımı (bağlı genler): Bir organizmadayüzlerce genin bulunması, kromozom sayısının daha azolması, tek kromozom üzerinde çok sayıda genin bulun-masını zorunlu kılar. Aynı kromozom üzerinde yerleşenbu genlere bağlı genler denir. Böylelikle insanda derirenginin kalıtımı çok genli kalıtıma örnektir.

64. İnsanda deri renginin kalıtımı aşağıdakilerdenhangisine bir örnektir?

A) Çok genli kalıtım

B) Pleiotropi

C) Epistaz

D) Eş baskınlık

E) Bağlantılı gen kalıtımı

Yanıt: A

Page 24: Ortaokul - Lise - ALTIN NOKTA · Endoplazmik Retikulumda sentezlenir C) Granüllü Endoplazmik Retikulum yapısında yer alan proteinlerin sentezi GER’da gerçekleşir D) Sitoplazmada

350

2012 - 20. Ulusal Biyoloji Olimpiyatı 1. Aşama Soruları ve Çözümleri

alt

ın n

ok

ta y

ay

ınla

rı ©

65. Bir tip cücelikte, çok parmaklılık yaygındır.Bunun nedeni aşağıdakilerden hangisidir?

A) Genlerin bağımsız ayrılması

B) Eş baskınlık

C) Çok genle kalıtılması

D) Ebeveynlerin heterozigot olması

E) Genlerin bağlantılı olması

ÇÖZÜM:

Verilere göre A ve D en uzağa yazılır.

B-A arası 1,5

B-C arası 4 olduğundan

A-C arası 2,5

C-D arası 3,5 bulunur.

en uzakta B ve D olduğuna göre bu iki genin koparakkarşı kromozoma geçme olasılığı en yüksektir.

4 3,5

B A C D

1,5 6

2,5

67. Aynı kromozomda A, B, C ve D adında 4 gen yeralmaktadır. Bu genlerin bazıları arasındaki rekombi-nasyon frekanslarından hesaplanan uzaklıklar aşağı-da verilmiştir.

B – A = 1.5

B – C = 4.0

A – D = 6.0

C – D = 3.5

Yukarıdaki verilere göre bu genlerin doğru sırasıaşağıdakilerden hangisidir?

A) A-B-C-D B) B-A-C-D C) A-B-D-C

D) B-C-A-D E) A-C-D-B

66. Bağlantılı iki gen açısından heterozigot olan(AaBb) bireyin, eşit olasılıklarla 4 gamet (AB,Ab, aB, ab) meydana getirebilmesinin enolası nedeni nedir?

A) Genler aynı homolog kromozom üzerinde bulunur

B) Sperm yada yumurtada ayrılmama olayı meydanagelmiştir

C) Genler homolog kromozomlar üzerinde değildir

D) Aynı kromozom üzerindeki genlerin arasındakimesafe çok uzaktır

E) Aynı kromozom üzerindeki genler birbirlerineyakındır

ÇÖZÜM:

* Dengeleyici seçilim: Uçtaki ekstrem fenotiplerekarşı ortada yer alan varyasyonları koruyandoğal seçme şeklidir.

* Suni seçilim: İnsan eliyle yapılan seçilimdir.

* Yönlü seçilim: Fenotipik skalanın bir ucundabulunan bireyleri koruyan doğal seçmedir.

* Normalleştirici seçilim: Türün standart morfoti-pinden bir sapma olduğunda bunu ortadan kaldı-ran seçilimdir.

* Dallandırıcı seçilim: Bir özelliğin ekstrem ucunasahip olan bireyler daha yüksek uyum gücünesahip olduğunda ortaya çıkar. Populasyondafenotipik varyasyonların artışı ile sonuçlanabilir.

68. Populasyonun normal dağılımında (çan eğrisi)en uç karakterlerin tercih edilip ortalamadakibireylerin populasyondan elimine olmasınaneden olan seçilim tipi aşağıdakilerden han-gisidir?

A) Dengeleyici seçilim B) Suni seçilim

C) Yönlü seçilim D) Dallandırıcı seçilim

E) Normalleştirici seçilim

ÇÖZÜM:

Cüce olan birinin aynı zamanda çok parmaklı olması,cüceliğin aktarımı sırasında çok parmaklılığında akta-rılması bu iki genin bağlı olduğunu düşündürür.

ÇÖZÜM:

AaBb genotipindeki bireyde iki gen de aynı homologkromozom çifti üzerinde bağlantılı olduğu halde AB,Ab, aB ve ab şeklinde dört çeşit gamet oluşmasıKrossing over gerçekleştiğini gösterir.

İki gen arasındaki uzaklık ne kadar fazla ise bu genle-rin koparak karşı kromozoma geçme olasılıkları da okadar fazladır.

Page 25: Ortaokul - Lise - ALTIN NOKTA · Endoplazmik Retikulumda sentezlenir C) Granüllü Endoplazmik Retikulum yapısında yer alan proteinlerin sentezi GER’da gerçekleşir D) Sitoplazmada

351

2012 - 20. Ulusal Biyoloji Olimpiyatı 1. Aşama Soruları ve Çözümleri

alt

ın n

ok

ta y

ay

ınla

rı ©

69. Aşağıdakilerden hangisi zigot öncesi izolas-yon mekanizmalarından birisi değildir?

A) Hibrit kırılması B) Gamet izolasyonu

C) Habitat izolasyonu D) Mekanik izolasyon

E) Davranış izolasyonu

ÇÖZÜM:

Kökeni aynı görevi farklı ya da aynı olan organlarhomolog organlardır. Homolog organlardaki damar,sinir, kemik sayısı ve kemik dizilişi aynıdır.

Kelebek kanadı; kuş kanadı, yarasa kanadı, penguenyüzgeci ve insan kolu ile farklı kökenden gelir.

70. Aşağıdakilerden hangisi homoloji göster-mez?

A) Kuş kanadı B) Yarasa kanadı

C) Penguen yüzgeci D) Kelebek kanadı

E) İnsan kolu

ÇÖZÜM:

Tetrazomik bir insan hücresinin çekirdeğinde belli birhomolog kromozom çiftinden iki yerine dört tanebulunmasıyla açıklanır.

Ya da iki homolog kromozom çiftinin trizomisiyle orta-ya çıkar. Normal hücrede 46 kromozom olması gere-kirken 48 kromozom olur.

72. Tetrazomik bir insan hücresinin çekirdeğindekaç adet kromozom bulunur?

A) 46 B) 47 C) 48 D) 49 E) 50

ÇÖZÜM:

Ki-Kare uygunluk testi ile belirli bir değişkenin farklıkategorilerine ait gözlenen frekansların, beklenen fre-kanslarına uygunluğu araştırılır. Serbestlik derecesi(S.D)=n-1’den bulunur. n=grup (fenotip çeşit) sayısınaeşittir. Dihibrit çaprazlamada

AaBb x AaBb

Aa x Aa

AA Aa Aa aa

Fenotip = 2

Bb x Bb

BB Bb Bb bb

Fenotip = 2

Fenotipçeşidi

= 2.2 = 4

n = 4 S.D = (n–1)S.D = 4–1S.D = 3

71. Dihibrit bir çaprazın sonuçlarının Mendel kuralla-rına uyup uymadığını test etmek için Ki-kareanalizi yapılacaktır. Bu analizde serbestlikderecesinin kaç alınması gerekir?

A) 4 B) 3 C) 2 D) 1 E) 0

73. Aşağıdaki soyağacını en iyi açıklayan kalıtımmodeli hangisidir?

A) Otozomal baskın B) X-bağlantılı baskın

C) Otozomal çekinik D) X-bağlantılı çekinik

E) Otozomal eş baskın

ÇÖZÜM:

Hibrit kırılması: Farklı türlerin çiftleşmesinden oluşanbireyler verimli olabilirken bundan sonraki kuşaklardaçok zayıf ve kusurlu bireylerin oluşmasıdr.

Gamet İzolasyonu: Gametlerin birbirini tanımasıyumurtayı saran kılıf üzerinde yer alan özgül molekül-lerin varlığına bağlıdır. Bu özgül moleküller sadeceaynı türün sperm hücreleri üzerinde yar alan uygunmoleküllere tutunur.

Habitat İzolasyonu: Aynı alan içerisinde farklı habitat-larda yaşayan iki tür teknik açıdan coğrafik olarak yalı-tılmış olmamalarına karşın birbirleriyle nadiren karşıla-şıyor olabilirler. Biri suda biri karada yaşayan iki yılantüründe olduğu gibi.

Mekanik İzolasyon: Birbirlerine yakın akraba olan tür-ler çiftleşme girişiminde bulunabilirler fakat anatomikolarak birbirlerine uygun olmadıklarından çiftleşmeolayını tamamlayamazlar.

Davrenış İzolasyonu: Eşleri cezbeden özel işaretlerbir türe özgü özel davranışlar olduğu için birbirineyakın akraba türler arasında en önemli üreme engelinioluştururlar. Hibrit kırılmasında zigot oluşuyor veüreme gerçekleşiyor.

Diğer durumlarda ise zigot oluşamıyor.

Yanıt: A

Page 26: Ortaokul - Lise - ALTIN NOKTA · Endoplazmik Retikulumda sentezlenir C) Granüllü Endoplazmik Retikulum yapısında yer alan proteinlerin sentezi GER’da gerçekleşir D) Sitoplazmada

352

2012 - 20. Ulusal Biyoloji Olimpiyatı 1. Aşama Soruları ve Çözümleri

alt

ın n

ok

ta y

ay

ınla

rı ©

ÇÖZÜM:

Verilen soyağacındaki fenotip dağılımını en iyi açıkla-yan kalıtım modeli otozomal çekinik olma durumudur.Çünkü ana baba fenotipinde gözlenmeyen bir özellikoğul dölde ortaya çıkmıştır. Bu bireyin kız olması xbağlantılı çekinik model olmadığını gösterir. Otozomalbaskın ve x bağlantılı baskın özellik anne ya da baba-da gizli kalamaz.

74. I. Seçilim olmaması

II. Mutasyon olmaması

III. Göç olmaması

IV. Gen akışı olması

V. Çok büyük populasyon olması

VI. Genetik sürüklenme olması

VII. Rastgele eşleşme olması

VIII. Kurucu etkisinin olması

IX. Darboğaz etkisi olması

Bir populasyonda genetik dengenin sağlan-ması için yukarıdakilerden hangilerinin yeri-ne getirilmesi gerekir?

A) I-II-III-V-VII B) I-III-IV-V-VI-VII

C) II-III-IV-V-VI-VIII D) II-III-IV-V-VI

E) I-II-V-VI-VII-IX

ÇÖZÜM:

Dört karakter açısından melez olan tetrahibrit iki bireyarasında yapılan bir çaprazda 81 farklı genotip oluşur.

AaBbCcDd x AaBbCcDd

Her bir bağımsız karakterin çaprazından üç çeşitgenotip çıkar.

3 x 3 x 3 x 3 = 81 çeşit

75. Tetrahibrit bireyler arasındaki bir çaprazda mey-dana gelebilecek farklı genotip sayısı nedir?

A) 3 B) 9 C) 27 D) 81 E) 243

ÇÖZÜM:

Üreme izolasyonu: Kendi aralarında üreyebilen birgrubun herhangi bir nedenden ötürü birbirleriyle olanilişkilerinin kesintiye uğramasıdır. Bu durum yeni tür-lerin oluşmasına ve türleşmeye neden olur.

Doğal seçilim: Üreyen canlı nüfusunda, yararlı ırsiözelliklerin nüfusta görülme sıklığının sonraki nesiller-de arttığı, zararlı ırsi özelliklerin nüfusta giderek azal-dığı süreçtir. (Üreme izolasyonu evrimi hızlandırır.)

Gen akışı: Genlerin bir populasyondan diğerine hertürlü hareketidir. İzolasyonun olmaması demektir.(Üreme izolasyonunun evrimini yavaşlatır.)

Mutasyonlar: Geçirerek ayıklanma (seçilim) evrimgeçirmiş bir türün oluşum sürecini hızlandırabilir.(Üreme izolasyonu evrimini hızlandırır.)

Genetik sürüklenme: Gen frekansları bakımından aitolduğu populasyonda önemli ölçüde farklı olan küçükbir birim populasyondan ayrılıp yeni bir populasyonkurucusu olarak görev yapabilir. Zaman içinde yenipopulasyonların oluşmasına neden olur. Rastgele çift-leşme üreme izolasyonunun evrimini hızlandırıcı etkigöstermektedir.

77. Aşağıdaki evrimsel mekanizmalardan hangisiüreme izolasyonunun evrimini yavaşlatır yada engeller?

A) Doğal seçilim B) Gen akışı C) Mutasyon

D) Genetik sürüklenme E) Rastgele çiftleşme

ÇÖZÜM:

Populasyonda toplam 1000 allel gen bulunur.Bunlardan 800 ü A, 200 ü a dır.

1000 allelden 800 ü A ise

100 allelden 80 i A dır.

76. Bir populasyonda AA genotipli 350, Aa genotipli100 ve aa genotipli 50 adet birey varsa A alelininbu populasyondaki frekansını hesaplayınız?

A) 0.7 B) 0.8 C) 0.9 D) 0.2 E) 0.32

ÇÖZÜM:

Bir populasyonda genetik dengenin sağlanabilmesi için;

- Göç

- İzolasyon

- Mutasyon

- Seleksiyon ve

- Rastgele olmayan çiftleşmeler olmaması

gerekir. Bunlar olursa gen frekansları değişir.

Bir populasyondan dışarı gen akışı olması, kurucuetkisi ve darboğaz etkisi olması genetik dengeyi bozar.

Yanıt: B

Page 27: Ortaokul - Lise - ALTIN NOKTA · Endoplazmik Retikulumda sentezlenir C) Granüllü Endoplazmik Retikulum yapısında yer alan proteinlerin sentezi GER’da gerçekleşir D) Sitoplazmada

353

2012 - 20. Ulusal Biyoloji Olimpiyatı 1. Aşama Soruları ve Çözümleri

alt

ın n

ok

ta y

ay

ınla

rı ©

ÇÖZÜM:

M bölgesinde L bölgesine göre daha fazla tür kaybıolmasının nedeni yosunlu alanlar arasındaki toplambüyüklük farkı olamaz. Bunun esas nedeni türlerinküçük alanlara hapsedilmiş ve yaşam alanlarının da-raltılmış olmasıdır.

Alan daraldıkça rekabet artmış ve bazı türler ortamdanelenmiştir.

L bölgesinde yayılmayı sağlayan küçük koridorlarolması bazı türlerin hayatını kurtarmıştır.

78. Bir grup araştırmacı, üzeri yosun kaplı büyük birkayanın bazı yerlerini kazıyıp o bölgeleri yosun-suz bırakarak birbirinden ayrı kalmış mikrohabi-tat yamaları (K, L, M) oluşturdular. Bu yosunlubölgelerde aktif olarak uçma yeteneğine sahipolmayan küçük eklembacaklı (mikro-arthropod)türlerini izlemeye başladılar.

Çalışma başlangıcından 6 ay sonra K bölgesi ilekıyaslandığında; L bölgesindeki eklembacaklıtürlerinin % 14’ünün, M bölgesinde ise % 41’ininyok olduğu gözlenmiştir.

Bu çalışmayla ilgili olarak;

I. M bölgesinde, L bölgesine göre daha fazla türkaybı olmasının nedeni, iki habitat yamasındakiyosunlu alanlar arasındaki toplam büyüklük far-kıdır,

II. Birbirinden yalıtılmış daha küçük habitat yama-larında tür çeşitliliği, büyük yamalara göre daha

hızlı azalmaktadır,

III. Habitat yamaları arasında bireylerin yayılmasınıdestekleyen koridorlar, küçük habitatlarda türle-rin yok olma hızını önleme yönünde kurtarmaetkisi yaratmaktadır,

şeklindeki yargılardan hangisi veya hangileridoğrudur?

A) Yalnız I B) Yalnız II C) Yalnız III

D) I ve III E) II ve III

Koridorlar 10 mm likparçalarla birbirindenayrılıyor.

Kesiler 7 cm likkoridorlarla birbirinebağlanıyor.

Kontrol parçası

50cm

50cm

(M Bölgesi)(L Bölgesi)(K Bölgesi)

79. Aşağıdaki şemada yıllık sıcaklık ve yağış ortalama-larına göre karasal biyomların dağılışı verilmiştir.

I, II ve III ile gösterilen biyomlar, tür çeşitliliğien yüksek olandan en düşük olana doğrunasıl sıralanır?

A) I, III, II B) II, I, III C) III, II, I D) II, III, I E) III, I, II

III

II

I

400

300

200

100

0-10 0 10 20 30

Yıllık sıcaklık ortalaması (°C)

Yıllık

yağ

ış(c

m)

ÇÖZÜM:

Grafikte II ile gösterilen kısım en çok yağışı alan ve yıl-lık sıcaklık ortalaması 30ºC civarında olan bir biyom-dur. Burada tür çeşitliliği çok fazla olur. III ile gösterilenkısım çölleri oluşturur. Burada hem hayvanların hembitkilerin kuraklığa dayanıklı olanları bulunur. II ile gös-terilen bölgede yılın uzun bir bölümünde toprak buzlakaplı olduğundan soğuk sınırlayıcı bir faktördür.Burada yaşayabilen bitki ve hayvan çeşidi diğerbiyomlara göre en azdır.

80. Bir araştırmacı, Kuzey Amerika’nın batısındakikayalık kıyı bölgesinde (intertidal zon) komüniteyapısını incelemiştir. Bu bölgede denizyıldızları,midyelerin başlıca avcılarıdır. Denizyıldızınınolmadığı durumlarda, midyeler kayalık zeminintamamına yayılarak ortamı kendi denetimi altınaalmışlardır. Denizyıldızları ortamda mevcut oldu-ğunda ve deneysel olarak uzaklaştırıldığındakomünitenin tür çeşitliliği grafikteki gibi ifadeedilmiştir.

Yıl

1963 64 65 66 67 68 69 70 71 72 73

20

15

10

5

Mevcu

ttü

rsayıs

ı

Deniz yıldızı mevcut

Deniz yıldızı deneyselolarak uzaklaştırılmış

Page 28: Ortaokul - Lise - ALTIN NOKTA · Endoplazmik Retikulumda sentezlenir C) Granüllü Endoplazmik Retikulum yapısında yer alan proteinlerin sentezi GER’da gerçekleşir D) Sitoplazmada

354

2012 - 20. Ulusal Biyoloji Olimpiyatı 1. Aşama Soruları ve Çözümleri

alt

ın n

ok

ta y

ay

ınla

rı ©

Bu çalışma kapsamında,

I. Denizyıldızları, bu bölgedeki komünitede kilittaşıtürdür,

II. Midyeler, denizyıldızlarıyla rekabet halindedir,

III. Midyelerin baskın olduğu durumlarda komünite-de tür çeşitliliği artmaktadır,

şeklindeki yargılardan hangisi veya hangile-rine ulaşılabilir?

A) Yalnız I B) Yalnız II C) Yalnız III

D) I ve II E) I ve III

ÇÖZÜM:

Los Hermanos ve Daphne adalarında yaşayan popu-lasyonlar bir araya getirilirse aralarında beslenmerekabeti yaşanabilir. Çünkü gaga uzunluğu aynı olanbirey sayısı her iki populasyonda da fazladır.

San Cristobal adasında yaşayan populasyonlardagaga yapıları farklı büyüklüklerdeki besinlerle beslen-mek üzere farklılaşmıştır. Bu türler arasında bu yüzdenbeslenme rekabeti olmaz.

G. Fortis tek başına bulunduğu durumda G. Fliginosaile bir arada bulunduğundan daha büyük yapılı besin-lerle beslenmeye uyum sağlamamıştır. Çünkü birarada bulunduğu durumda gaga derinliği iyice artmış-tır. Yalnızken daha küçük besinlerle beslenmektedir.

81. Şekilde, tohumla beslenen Galapagos ispinozla-rından iki türün (Geospiza fuliginosa ve Geospizafortis) farklı adalarda ve aynı adada bulunanpopulasyonlarında gaga derinliği (genişliği) vebunların sayısal yüzdeleri verilmiştir.

Buna göre;

I. Los Hermanos ve Daphne adalarında yaşayanpopulasyonlar bir araya getirilirse aralarındabeslenme rekabeti yaşanabilir,

II. San Cristobal adasında yaşayan populasyonlar-da gaga yapıları farklı büyüklüklerdeki besinlerlebeslenmek üzere farklılaşmıştır,

III. G. fortis tek başına bulunduğu durumda, G. fuli-ginosa ile bir arada bulunduğundan daha büyükyapılı tohumlarla beslenmeye uyum sağlamıştır,

şeklindeki yargılardan hangisine veya hangi-lerine ulaşılabilir?

A) Yalnız I B) Yalnız III C) I ve II

D) II ve III E) I, II ve III

82. Aşağıdaki grafikte altı ülkenin 1970 ve 2006 yıl-larında kadın başına düşen ortalama çocuksayıları verilmiştir. Yatay kesikli çizgi, dünyanınuzun dönemde sıfır populasyon büyümesineulaşabilmesi için kadın başına düşen ortalamaçocuk sayısını göstermektedir.

Grafikteki verilere göre,

I. Üreme oranındaki en fazla azalma Çin’degörülmüştür,

II. Hindistan, otuz altı yıl sonra negatif büyümehızına ulaşmıştır,

III. ABD, Kanada ve Avustralya’nın üreme oranla-rındaki değişim, diğer üç ülkeden daha yavaştır,

şeklindeki yargılardan hangisine veya hangi-lerine ulaşılabilir?

A) Yalnız I B) Yalnız III C) I ve III

D) II ve III E) I, II ve III

Gagaderinliği

Gaga derinliği (mm)

Her

yükl

ük

sın

ıfın

dak

i bir

yeri

nyü

zdes

i

161412108

San Cristóbal adası60

40

20

0

G. fuliginosa veG. fortis

Daphne adası60

40

20

0

G. fortis

Los Hermano adası60

40

20

0

G. fuliginosa

G. fuliginosa G. fortis

AvustralyaKanadaABDÇinİran Hindistan

1970 verileri206 verileri

0

2

4

6

8

Kad

ınb

aşın

ad

üşen

ort

ala

ma

ço

cu

ksayıs

ı

ÇÖZÜM:

Kilittaşı tür: Doğal çevrelerinde sayılarına rağmenönemli etkide bulunan türlerdir. Bir ekosistemin içindenkilittaşı tür çekilirse bu tür biyokütle ya da verimlilik açı-sından küçük bir yer tutsa dahi ekosistemde çok drama-tik değişiklikler oluşabilir. Deniz yıldızları midyeleri avla-yarak midye populasyonunu kontrol altında tutarlar.Midye populasyonunun aşırı artmaması o çevredekibaşka türlere de yaşama olanağı sağlar. Kilittaşı tür olandenizyıldızları ortamdan uzaklaştırılırsa midyelerin aşırıartmasıyla diğer türler rekabete dayanamaz ve bir birortamdan yok olurlar. Midyeler denizyıldızlarının avıdır.

Page 29: Ortaokul - Lise - ALTIN NOKTA · Endoplazmik Retikulumda sentezlenir C) Granüllü Endoplazmik Retikulum yapısında yer alan proteinlerin sentezi GER’da gerçekleşir D) Sitoplazmada

355

2012 - 20. Ulusal Biyoloji Olimpiyatı 1. Aşama Soruları ve Çözümleri

alt

ın n

ok

ta y

ay

ınla

rı ©

ÇÖZÜM:

Grafikteki verilere bakılarak üreme oranının en fazlaazaldığı ülkenin Çin olduğu söylenebilir. Hindistandabir kadının doğurduğu çocuk sayısının üçe düşmesibüyüme hızının hala negatif bir değere düşmediğinigöstermektedir. ABD, Kanada ve Avustralya’da diğerülkelere göre üreme hızındaki düşmenin yavaş olmasıdoğaldır. Zaten bu ülkelerde artış oldukça azdır.

ÇÖZÜM:

Grafiğe göre; gövde biyomasının artış gösterdiği bütünyaşlarda yıllık net birincil üretim miktarları artma gös-termemiştir. Yaklaşık 70 yaşından sonra azalma gös-termektedir. Gövde biyoması ise yaklaşık 130 yaşınakadar artma, daha sonra azalma göstermektedir.

83. Şekilde Rusya ormanlarındaki ladin ağaçlarındayaş ile gövde biyoması (çizgi grafik) ve yıllık netbirincil üretim (çubuk grafik) miktarları arasında-ki ilişkiler verilmiştir.

Grafiğe göre,

I. Gövde biyomasının artış gösterdiği bütün yaş-larda, yıllık net birincil üretim miktarları da art-mıştır,

II. Yıllık net birincil üretim yaklaşık 70 yaşına kadarartma, sonrasında azalma göstermektedir,

III. Gövde biyoması yaklaşık olarak 130 yaşınakadar artma, sonrasında azalma göstermektedir,

şeklindeki yargılardan hangisine veya hangi-lerine ulaşılabilir?

A) Yalnız I B) Yalnız II C) Yalnız III D) I ve II E) II ve III

200

160

120

80

40

00 20 40 60 80 100 120 140

0

1

2345

678

Yaş (yıl)

vd

eb

iyo

ması(t

on

/ha) Y

ıllıkn

et

birin

ciü

retim

(ton

/ha/y

ıl)

Gövde biyoması

84. Grafikte beş yıllık süre içinde Antarktik fin balina-sı populasyonundaki bazı özelliklerin değişimimodellenmiştir.

Grafiğe göre,

I. Model, maksimum elde edilebilecek ürünü,populasyon taşıma kapasitesinin %80-85’ineulaştığı durumda öngörmektedir,

II. Model, populasyonun %100 taşıma kapasitesi-ne ulaştığında en verimli avlanılabilecek stokbüyüklüğünü 11 bin birey olarak öngörmektedir,

III. Model, populasyonun %50 taşıma kapasitesiniyaklaşık 200 bin birey olarak öngörmektedir,

şeklindeki ifadelerden hangisi veya hangileridoğrudur?

A) Yalnız I B) Yalnız II C) Yalnız III

D) I ve III E) II ve III

Beş yıllık stok büyüklüğü (bin)

Avl

anan

stok

(bin

)

14

12

10

8

6

4

2

00 100 200 300 400

% 100% 0Taşıma kapasitesi

% 50

ÇÖZÜM:

Grafiğe göre; populasyon taşıma kapasitesinden% 80 - 85’ine ulaştığı durumda elde edilecek ürünmaksimumdur. Populasyon % 100 taşıma kapasitesi-ne ulaştığında en verimli avlanabilecek stok büyüklüğü11000 birey değil 4000 birey civarındadır. Modele görepopulasyon 200 bin bireye ulaştığında % 50 taşımakapasitesine de ulaşır.

Page 30: Ortaokul - Lise - ALTIN NOKTA · Endoplazmik Retikulumda sentezlenir C) Granüllü Endoplazmik Retikulum yapısında yer alan proteinlerin sentezi GER’da gerçekleşir D) Sitoplazmada

356

2012 - 20. Ulusal Biyoloji Olimpiyatı 1. Aşama Soruları ve Çözümleri

alt

ın n

ok

ta y

ay

ınla

rı ©

85. Yaban domuzları ile firavun fareleri aynı yaşamalanlarında birlikte bulunabilirler. Yaban domu-zunun vücudu üzerinde çok sayıda kene bulun-maktadır. Yaban domuzu bazen sakin bir şekildedurarak vücudunu uygun konuma getirdiğindefiravun faresi yaban domuzunun üzerindekikeneleri yemektedir. Bu üç canlı arasındakiyaşam şekilleri ile ilgili olarak aşağıda yapı-lan eşlemelerden hangisi doğrudur?

87. Hayvanlarda vücut büyüklüğü ile metabolizmahızı arasında bir ilişki vardır. Buna göre;

grafikte I, II ve III ile gösterilen hayvanlar için aşa-ğıda yapılan eşlemelerden hangisi doğrudur?

ÇÖZÜM:

Sınırlayıcı faktörlere karşı geniş tolerans gösteren tür-lerin yaşama ve üreme şansı daha fazla olduğundanfarklı yaşam alanlarına yayılım gösterebilirler. Üremeevresinde olan bireylerin tolerans aralığı daralır.

Bir ortamda ekolojik faktörlerden herhangibiri optimumdeğerde değilse canlının tolerans aralığı başka ekolo-jik faktörlere karşı da azalabilir.

86. Tolerans (hoşgörü) yasasına göre, çevresel fak-törler için her canlı türünün hoşgörü ile karşıla-yabileceği bir minimum ve maksimum sınır var-dır. Tolerans sınırları denilen bu iki sınır arasın-da kalan aralığa da tolerans aralığı adı verilir.

Buna göre,

I. Sınırlayıcı faktörlere karşı geniş tolerans göste-ren canlı türleri, genellikle daha geniş alanlardayayılabilme özelliği gösterirler,

II. Üreme evresinde olan bireylerin tolerans aralığı,üreme evresinde olmayan bireylerinkine göredaha geniş olur,

III. Bir canlı türü, bulunduğu ortamda ekolojik faktör-lerden herhangi biri için optimum koşullara sahipdeğilse, bu türün aynı ortamda başka ekolojikfaktörlere karşı tolerans aralığı azalabilir,

şeklindeki çıkarımlardan hangisi veya hangi-leri doğrudur?

A) Yalnız I B) Yalnız II C) I ve III

D) II ve III E) I, II ve III

ÇÖZÜM:

Memelilerde vücut büyüklüğü ile metabolizma hızı ara-sındaki ilişki grafiği;

Kuşlar yüksek vücut sıcaklığının sıklıkla gözlendiğicanlılardır. Bu sayede onlara uçarken ağırlık yaparaksorun yaratabilecek sudan kurtuldukları gibi, sıcaklığınsağladığı yüksek metabolik hız sayesinde de kaslarınıen verimli şekilde çalıştırılabiliyorlar. Gelişmiş canlıvarlıklarda ısı kaybı en düşük düzeydedir. Soğukkanlıcanlılarda metabolizma yavaş olduğundan fazla ısıüretilmez. 1000 g vücut ağırlığında metabolizma hızıen yüksek olan Kuş > memeli > Sürüngen şeklindesıralanır.

Vücut büyüklüğü

Metabolizma hızı

Vücut büyüklüğü azaldıkçametabolizma hızlanır.

Yaban domuzu ile firavun faresi arasında

Kene ile yabandomuzu arasın

dafaresi arasında

Firavun faresi ile kene

arasında arasında

A) Protokoopersyon Parazitlik Predatörlük

B) Komensalizm Nötralizm Amensalizm

C) Protokoopersyon Parazitlik Nötralizm

D) Nötralizm Komensalizm Predatörlük

E) Mutualizm Komensalizm Protokoopersyon

Vücut ağılığı (g)

Me

tab

olik

hız

(KJ/

n)

100.000

10.000

1.000

100

10100.00010.0001.00010010

III

I

II

I II III

A) Kuşlar Memeliler Sürüngenler

B) Memeliler Sürüngenler Kuşlar

C) Kuşlar Sürüngenler Memeliler

D) Sürüngenler Memeliler Kuşlar

E) Sürüngenler Kuşlar Memeliler

ÇÖZÜM:

İki canlının birbirine karşılıklı yarar sağladığı ve zorun-lu olmayan ilişki şekli protokooperasyon’dur. Yabandomuzunun kenelerini yiyen firavun fareleri yabandomuzunu parazitlerden kurtarır. Bir canlı bir canlıdanyararlanırken ona zarar verirse ilişki parazitliktir. Keneyaban domuzundan yararlanırken zarar verir.

Bir canlı avını canlı olarak tutup yerse ona predatördenir. Firavun faresi kenenin predatörüdür.

Page 31: Ortaokul - Lise - ALTIN NOKTA · Endoplazmik Retikulumda sentezlenir C) Granüllü Endoplazmik Retikulum yapısında yer alan proteinlerin sentezi GER’da gerçekleşir D) Sitoplazmada

357

2012 - 20. Ulusal Biyoloji Olimpiyatı 1. Aşama Soruları ve Çözümleri

alt

ın n

ok

ta y

ay

ınla

rı ©

ÇÖZÜM:

Kuyruk sallama dansı uzaktaki besinin yerini bulmakiçin yapılır. Çemberin kesiştiği doğrultu kovan ile besinve güneş arasındaki açıları gösterir. Dansın düz olarakverilen doğrultusu, yerçekimi doğrultusu ile 90 derece-lik açı yapıyorsa, bu yiyecek kaynağının Güneş’in 90ºsağında olduğunu gösterir.

88. Bulduğu bir besin kaynağından kovana geridönen bir bal arısı düşey düzlemde kuyruk salla-ma dansı yapmaktadır. Bu sırada tam sağ yuka-rıya yönelmektedir. Bu durumda besin kayna-ğının yeri aşağıdakilerden hangisidir?

A) Kovandan çıkınca güneşin 90º sağında

B) Kovandan çıkınca güneşin 90º solunda

C) Kovanın önünde ve 40º sağında

D) Kovana çok yakın bir yerde

E) Kovandan çıkınca güneşin 40º solunda

89. Aşağıdaki terimlerden hangisi hayvanlardaçeşitli koşullar altında yavruların kendi ebevyn-leri tarafından yenmesi durumunu ifadeeder?

A) Predasyon B) Kronizm C) Hipertrofizm

D) Agresiflik E) Agregasyon

ÇÖZÜM:

Predasyon: Av-avcı ilişkisidir. İki tür arasındaki ilişkiörneğidir. Bir tür diğer türü avlayarak yaşar. Üzerindezararlı etki yapar. Bir tür diğerini öldürür ya da yer.

Agregasyon: Çok çeşitli düşmanca davranışlar için kul-lanılan bir terimdir. Sosyete teşkil etmeyen bir türünbireylerinin bir yerde toplanmasıdır. Toplanarak biraraya gelme, kümeleşme anlamına gelir. Saldırı canlılararasında beslenme, çiftleşme, yuva alanını koruma,savunma, korku veya rekabetle ilgili davranışlardır.

Anne babaların yavrularını yemesine kronizm denir.

ÇÖZÜM:

Bir hayvanın tehlikeye karşı savunmaya hazır şekildebulunmasına; boynuzlar, hassas duyu organları, çevreile uyumlu post rengi ve hayvanın uzun süre hareket-siz kalarak bekleyebilmesi katkıda bulunur. Bunlarınhepsi pirimer savunma araçlarıdır. Hayvanın hızlakoşarak uzaklaşması sekonder bir savunma aracıdır.

90. Ortamda bir tehlike olup olmadığına bakmaksı-zın bir hayvanın savunmaya ait gereçlerininsürekli hazır olması haline primer savunmaaraçları denir. Bu tanıma göre aşağıdakilerdenhangisi geyiklerde bir primer savunma aracıdeğildir?

A) Boynuzlar

B) Hassas duygu organları

C) Çevre ile uyumlu post rengi

D) Hayvanın hızla koşarak uzaklaşabilmesi

E) Hayvanın uzun süre hareketsiz kalıp bekleyebilmesi

ÇÖZÜM:

Aşırı sevinç, korku, heyecan gibi durumlarda hayvanagüç ve performans kazandıran hormon adrenalindir.Ruhsal ve bedensel yüklenmelerde hayvan ya kaça-cak ya kovalayacak ya da savaşacaktır.

Adrenalin solunum, dolaşım, tansiyon ve kalp atımhızını artırır, karaciğerden kana glikoz akışını hızlandı-rır, kanın pıhtılaşma süresini de düşürür.

Progesteron dişide gebeliğin devamını sağlar.

Prolaktin korpus luteumun devamını sağlar, analıkiçgüdüsünü oluşturur, süt bezlerini geliştirir.

Testosteron erkeklik hormonudur. Spermatogenezindevamını sağlar ve erkekte ikincil cinsiyet özelliklerinioluşturur.

Östrojen dişilik hormonudur. Dölyatağının kalınlaşma-sını sağlarken dişide ikincil cinsiyet özelliklerini deoluşturur.

91. Aşağıdaki hormonlardan hangisi insanlardakaçma ve saldırı davranışları ile doğrudan ilgi-lidir?

A) Progesteron B) Adrenalin C) Prolaktin

D) Testosteron E) Östrojen

Page 32: Ortaokul - Lise - ALTIN NOKTA · Endoplazmik Retikulumda sentezlenir C) Granüllü Endoplazmik Retikulum yapısında yer alan proteinlerin sentezi GER’da gerçekleşir D) Sitoplazmada

358

2012 - 20. Ulusal Biyoloji Olimpiyatı 1. Aşama Soruları ve Çözümleri

alt

ın n

ok

ta y

ay

ınla

rı ©

ÇÖZÜM:

(Gymnospermar) açık tohumlularda polen kozalakları-nın üzerinde mikrosporangium içerisinde diploit mik-rospor ana hücreleri bulunur. Bunlar mayozla sporlarıoluşturur. Polen taneleri erkek gametofitlerdir. Poleninzarımsı kanatları vardır. Polen ovül üzerine yapıştıktansonra polen tüpü oluşur ve spermler yumurtaya taşınır.Ancak bundan sonra gelişme çok yavaş gerçekleşir.Bir ay sonra megaspor ana hücreleri mayoz geçirir.Bunlardan sadece bir tanesi olgun bir dişi gametofitolacak şekilde gelişir. Tozlaşmadan 15 ay sonra spermyumurtayı döller. Ancak bir embriyo tam olarak gelişe-bilir. Tohumlarda biriktirilen besin dişi gametofitin hap-loit dokusunu oluşturur. Tohum taslakları dişi gametofi-tin içine gömülmüştür. Tek döllenme görülür ve endos-perm n kromozomludur.

94. Aşağıda I, II, III ve IV ile verilen özelliklerin han-gisi veya hangileri Gymnospermae (Açıktohumlular) bitki grubu için doğrudur?

I. Tohumunda biriktirilen besin, dişi gametofitinhaployit dokusunu oluşturur

II. Tohum taslakları, dişi sporofitin içine gömülmüş-tür

III. Çift döllenme görülür

IV. Polenleri kanatsızdır

A) Yalnız II B) Yalnız I C) I, II ve III

D) I ve II E) III ve IV

ÇÖZÜM:

Kaslı diyafram sadece memelilerde

Dört odalı kalp kuş ve memelilerde

Kıl sadece memelilerde

Tüy sadece kuşlarda

Amniyotik yumurta sürüngen, kuş ve memelilerde

bulunur.

95. Murat, bir kâğıda günümüz omurgalı hayvanla-rında bulunan bazı özellikleri listelemiş ve arka-daşı Volkan’a vermiştir. Bu özelliklerden tekbaşına bir omurgalı hayvan sınıfını belirleyenleriişaretlemesini istemiştir. Listedeki özellikler;

I. Kaslı diyafram

II. Dört odalı kalp

III. Kıl

IV. Tüy

V. Amniyotik yumurta

olduğuna göre, Volkan hangilerini işaretlerseMurat’ın sorusunu tam olarak doğru yanıtla-mış olacaktır?

A) I ve III B) II ve V C) I, III ve IV

D) I, IV ve V E) III, IV ve V

ÇÖZÜM:

Atkuyruklarında ekuisetum denilen toprak altı rizomla-rından dikey gövdeler çıkar. Dik ve içi boş gövdelerboğumludur. Bu boğumlardan halkalar oluşturan yap-raklar çıkar. Ekuisetumun bazı gövdelerinin ucundasporangiumları taşıyan kozalak benzeri yapılar bulu-nur. İletim dokusu bulundururlar fakat mega yapraklarıyoktur, sporları kamçılıdır. Gerçek kök ve yapraklarıvardır.

92. Aşağıda I, II, III ve IV ile verilen özelliklerin han-gisi veya hangileri Sphenopsida (atkuyrukla-rı) grubuna aittir?

I. İletim dokusu var

II. Mega yapraklar var

III. Kamçısız spor hücreleri var

IV. Gerçek kök ve yaprakları yok

A) II ve III B) Yalnız III C) Yalnız I

D) II ve IV E) II, III ve IV

ÇÖZÜM:

Karayosunlarında ergin bitkiler gametofittir. Dişi game-tofit üzerinde dişi organ ve dişi organda yumurta hüc-resi oluşur. Erkek gametofitten gelen spermle döllenenyumurta zigotu, zigot da saprofiti oluşturur. Sporofittemayozla oluşan sporlar çimlenerek yeni gametofitlerioluşturur.

Şekilde verilen kısımlardan;

I- Kaliptra

II- Sporofit

III- Gametofit

IV- Kapsül

dür.

93. Karayosununa ait aşağıdaki şekil üzerinde I, II,III ve IV ile gösterilen bölgeler hangi seçenektedoğru olarak verilmiştir?

IIIII

IIV

I II III IV

A) Kaliptra Gametofit Sporofit Kapsül

B) Sprofit Kapsül Gametofit Sporofit

C) Kapsül Kaliptra Sporofit Gametofit

D) Gametofit Sporofit Kaliptra Kapsül

E) Kaliptra Sporofit Gametofit Kapsül

Page 33: Ortaokul - Lise - ALTIN NOKTA · Endoplazmik Retikulumda sentezlenir C) Granüllü Endoplazmik Retikulum yapısında yer alan proteinlerin sentezi GER’da gerçekleşir D) Sitoplazmada

359

2012 - 20. Ulusal Biyoloji Olimpiyatı 1. Aşama Soruları ve Çözümleri

alt

ın n

ok

ta y

ay

ınla

rı ©

96. Aşağıdakilerden hangisi Arthropoda (Eklembacaklılar) altşubeleri için ortak bir karakter-dir?

A) Açık dolaşım

B) Ventral sinir kordonu

C) Şizosöl sölom

D) Vücut segmentlerinin özelleşmesi

E) Deri değiştirme

ÇÖZÜM:

Böcek takımlarında kuşlarda ve yarasalarda olduğugibi kanat on üyenin değişmesiyle ortaya çıkmamıştır.Pterygota’da bazı gruplarında ve türlerinde kanat ikin-cil olarak kaybolmuştur. (Bitlerde olduğu gibi)Protoraksta bazı gruplarda kutikular gelişmeler olmak-la birlikte kanat görülmemiştir. Bazı türlerinde ise küçü-lerek pul şeklini almıştır. Örnek olarak brochyotera.(Kelebek, evcikli böcekler) (Paronota) Zar kanatlılardazar gibi ya da saçak gibi alt yanlarından dışa doğrugelişen vücut duvarı kıvrımları (Paronota) kanatlarımeydana getirir. Paronota çift duvarlıdır. Kanatlarınoluşumu çok büyük bir yayılma yeteneği kazandırmış-tır. Pasif yolları kullanarak geçmişte fosil örneklerindehep kanat oluşumu ortaya çıkmıştır.

97. Böcek takımlarının akrabalık durumlarını düşün-düğümüzde aşağıdaki karakterlerden hangisipirimitif (ilkel) ortak karakterdir?

A) Kanatların pullarla kaplı olması

B) Arka kanatların denge organına dönüşmüş olma-sı

C) Kanatlı olma durumu

D) Ön kanatların sekonder olarak kaybolmuş olması

E) Her iki çift kanadın da zar yapıda olması

SORU İPTAL EDİLDİ

98. Böcekleri dikkate aldığınızda, aşağıdakilerdenhangisi eşeysel dimorfizmin en yaygın görüle-bileceği vücut yapılarından birisidir?

A) Antenler

B) Ağız yapıları

C) İkinci çift bacaklar

D) Göğüsteki segment sayısı

E) Baştaki segment sayısı

ÇÖZÜM:

- Eşeysel dimorfizm: Erkek ve dişi eşeyler arasın-da vücut boyutu ve şekli, renk veya desen bakı-mından farklılıklar görülmesi durumudur.

- Böceklerde eşeysel dimorfizm: Antenlerde, ağızyapılarında, bacaklarda, göğüs ve baştaki seg-ment sayılarında görülebilir.

Böceklerde ANTEN tipleri;

1) Serrat

2) Filiform

3) Pektiat

4) Bipektinat

5) Pulmase

6) Genikulat

7) Lamellat

8) Aristat

olmak üzere 8 çeşittir.

Bu nedenle böceklerde eşeysel dimorfizm en yaygınantenlerde görülür.

Böceklerde AĞIZ tipleri ise;

1) Çiğneyici ağız

2) Delikli - emici ağız

3) Emici ağız

4) Yalayıcı-emici ağız

olmak üzere 4 çeşittir.

Böceklerde BACAK tipleri ise;

1) Koşucu-yürüyücü bacak

2) Sıçrayıcı bacak

3) Yakalayıcı bacak

4) Taşıyıcı-toplayıcı bacak

5) Kazıcı bacak

6) Yüzücü bacak

olmak üzere 6 çeşittir.

Göğüs bölgesindeki segment sayısı 3’tür.

1) Prothorax

2) Mesothorax

3) Metathorax

Baş bölgesindeki segment sayısının 5 (en fazla 6)olduğu varsayılmaktadır.

1) Preantenar

2) Antennar

3) İntercalar

4) Mandibular

5) Maksillar

6) Labial

Page 34: Ortaokul - Lise - ALTIN NOKTA · Endoplazmik Retikulumda sentezlenir C) Granüllü Endoplazmik Retikulum yapısında yer alan proteinlerin sentezi GER’da gerçekleşir D) Sitoplazmada

360

2012 - 20. Ulusal Biyoloji Olimpiyatı 1. Aşama Soruları ve Çözümleri

alt

ın n

ok

ta y

ay

ınla

rı ©

ÇÖZÜM:

Şekle göre vücut ağırlığı arttıkça trakeal alan artmak-tadır. İri vücut daha çok oksijene ihtiyaç duyduğu içinbu alan fazla olmalıdır.

Ortamın O2 yoğunluğunun az olması sonucu da bualan fazla olmalı ki gerekli oksijien alınabilsin.

ÇÖZÜM:

Dişi güvenin salgıladığı feromonlar dişi güveden uzak-laştıkça yayılır. Bu yüzden uzak kısımlarda erkekaşağı yukarı uçarak kokuyu ararken yaklaştıkça dahadüz bir hatta ilerler.

99. Aşağıda şematize edildiği gibi, böceklerde solu-num sistemi vücut hücrelerine doğrudan havataşıyan dallanmış iç borucuklardan (trake siste-mi) oluşmaktadır. Aşağıda gösterilen unkurdularvaları % 21, % 15, %10’luk farklı oksijenkoşullarında yetiştirilmiş ve “unkurdu larvasınınağırlığı” ile “her bir solunum deliğindeki trakealalan (mm2)” karşılaştırılmıştır.

Buna göre aşağıdaki sonuçlardan hangisininelde edilmesi beklenir?

A) Vücut ağırlığı arttıkça trakeal alan artar, düşükoksijen koşullarında yetiştirilen larvalarda bu alançok daha yüksektir

B) Vücut ağırlığı arttıkça trakeal alan azalır, düşükoksijen koşullarında yetiştirilen larvalarda bu alançok daha yüksektir

C) Yüksek oksijen oranlarında yetiştirilen larvalardatrakeal alan aynı ağırlıktaki düşük oksijen oranla-rında yetiştirilen larvalardan daha fazladır

D) Trakeal alan en fazla %15 oksijen oranında yetişti-rilen larvalarda fazladır

E) Trakeal alan farklı koşullarda değişmez, ancakvücut büyüklüğüne bağlı olarak artış gösterir

100. Güvelerde çiftleşmek için eş bulmada özellikleferomonlar kullanılır. Erkekler, oldukça uzakmesafelerden dişilerin yerlerini bu sayede bula-bilirler. Aşağıdakilerden hangisi erkeğin dişi-ye ulaşma yolunu betimler?

DIŞORTAM VÜCUT İÇ

Spiracle(Trake girişi)

Trake Trakeoller Doku

A)

B)

C)

D)

E)

DİŞİ

ERKEK

Rüzgar

DİŞİ

ERKEK

Rüzgar

DİŞİ

ERKEK

Rüzgar

DİŞİ

ERKEK

Rüzgar

DİŞİ

ERKEK

Rüzgar

Yanıt: B